Year 3 cases Flashcards

1
Q

A 57-year-old woman with a history of stage I breast cancer status post-lumpectomy is being treated with doxorubicin and cyclophosphamide every 3 weeks. She had her first treatment 8 days ago and now presents with a temperature of 38.2°C (100.8°F). Vital signs are stable and she has no localising signs or symptoms other than mouth sores. White blood cell count is 550/microlitre with an absolute neutrophil count of 200 cells/microlitre.
What is the diagnosis?

A

Neutropenic sepsis

How well did you know this?
1
Not at all
2
3
4
5
Perfectly
2
Q

A 40-year-old woman awakens with left-sided facial fullness and a subjective feeling of facial and tongue ‘numbness’ without objective hypoaesthesia. She also notes left-sided dysgeusia. Later that day she develops left-sided otalgia, hyperacusis, post-auricular pain, and facial discomfort. Left-sided facial palsy ensues, with associated oral incompetence, facial weakness, and asymmetry progressing to complete flaccid paralysis by the next morning. On physical examination, the resting appearance of the left face demonstrates brow ptosis, a widened palpebral fissure, effacement of the left nasolabial fold, and inferior malposition of the left oral commissure. There is complete absence of brow movement, incomplete eye closure with full effort, and loss of smile, snarl, and lip pucker on the affected side. The remainder of the history and physical examination are unremarkable.
What is the diagnosis?

A

Bells palsy

How well did you know this?
1
Not at all
2
3
4
5
Perfectly
3
Q

A 19-year-old man is diagnosed with a highly proliferating non-Hodgkin’s lymphoma. The disease is bulky, involving lymph nodes above and below the diaphragm, spleen, and bone marrow. Serum lactate dehydrogenase is significantly raised, but renal function and electrolytes are within normal limits. Twenty-four hours after initiation of aggressive chemotherapy he complains of nausea, vomiting, diarrhoea, and lethargy. He has become oliguric and is hypertensive and tachycardic. Biochemistry demonstrates elevated uric acid, potassium, and phosphate, as well as raised urea and creatinine.
What is the diagnosis?

A

TLS

How well did you know this?
1
Not at all
2
3
4
5
Perfectly
4
Q

A 69-year-old woman with a 2-year history of chronic lymphocytic leukaemia presents with a white blood cell (WBC) count of 41 x 10^9/L (41,000/microlitre). She has a past medical history of hypertension and mild renal impairment related to the use of NSAIDs for osteoarthritis. She is started on systemic chemotherapy with fludarabine. At follow-up 7 days after initiation of treatment she complains of fatigue and weakness. Her WBC count has fallen to within normal levels but serum biochemistry reveals hyperuricaemia, hyperkalaemia, hyperphosphataemia, hypocalcaemia, and a significant deterioration in renal function.

A

TLS

How well did you know this?
1
Not at all
2
3
4
5
Perfectly
5
Q

A 60-year-old man presents with progressive headache and cognitive decline. A MRI of the brain without gadolinium enhancement demonstrates a large extra-axial lesion that is similar intensity to brain on T1 images. After contrast administration, the lesion enhances avidly. The tumour was removed by craniotomy and the patient has been followed with serial imaging studies (once per year) with no evidence of recurrence after 6 years.

A

meningioma

How well did you know this?
1
Not at all
2
3
4
5
Perfectly
6
Q

A 52-year-old man presents with some difficulty driving at night and reports not seeing cars coming from the sides. He also describes progressive loss of libido and inability to obtain and maintain an erection, which started about 2 years ago. He reports bumping into things. He has gained about 5 kg (11 lb) in weight over the past 2 to 3 years. He has fatigue and is unable to do the same jobs that he used to do a year ago. The examination reveals moderate obesity (BMI 35) with some loss of muscle bulk over the proximal arm and leg muscle groups. Other positive findings include the presence of small bilateral gynaecomastia, soft testicles (12 mL), and abnormal visual fields to confrontation, with bitemporal hemianopia.

A

non functioning pituitary adenoma

How well did you know this?
1
Not at all
2
3
4
5
Perfectly
7
Q

A 40-year-old woman presents with a history of progressively decreased hearing in her left ear over the past few years. She noticed the hearing deficit when trying to use the phone with the left ear. She has recently complained of intermittent dizziness, tinnitus in the left ear, and vague left-sided headaches.

A

vestibular schwannoma

How well did you know this?
1
Not at all
2
3
4
5
Perfectly
8
Q

A 6-year-old boy presents with 3 to 4 weeks of morning headaches and intermittent vomiting without fever. The headaches improve throughout the day. Over the past 2 days, the headaches are lasting longer and the vomiting is more frequent, but after vomiting the headaches are much improved. The headaches are not localised to one side. On the day of presentation, the parents note that he is walking like a ‘drunken sailor’.

A

medulloblastoma

How well did you know this?
1
Not at all
2
3
4
5
Perfectly
9
Q

A 35-year-old right-handed man presents with a partial seizure involving jerky movements of his right arm and the right side of his face. He also reports left-sided headaches for a few months and clumsiness of his right hand. On examination he is awake and orientated. He has a subtle facial droop and pronator drift on the right side.

A

asytrocytic brain tumour

How well did you know this?
1
Not at all
2
3
4
5
Perfectly
10
Q

A 65-year-old woman presents with cognitive and memory changes over the past few weeks, associated with headaches over the past week. On examination she is apathetic and has slow mentation and left-sided weakness.

A

astrocytic brain tumour - glib

How well did you know this?
1
Not at all
2
3
4
5
Perfectly
11
Q

A 16-year-old girl presents with primary amenorrhoea, galactorrhoea, and mild headaches. Ophthalmological examination reveals loss of vision in the right eye (20/40).

Other presentations
Approximately 50% of patients present acutely with signs and symptoms of raised intracranial pressure (headache, nausea, vomiting, depressed sensorium, diplopia) or acute visual loss requiring emergent surgery. [2] [3] [4] Visual impairment is found preoperatively in at least 75% of patients, although this is uncommon as the only presenting symptom.

A

craniopharyngiomas

How well did you know this?
1
Not at all
2
3
4
5
Perfectly
12
Q

A 44-year-old male smoker presents with a 9-year history of recurrent headaches. Headaches occurred twice-monthly initially, always in the early hours of the morning (2 a.m. to 3 a.m.). The headaches have increased to an average of 2 episodes per day. The acute episodes can occur at any time, and last between 2 and 4 hours. He always has a nocturnal event. Attacks are triggered immediately after drinking alcohol or with the smell of strong aftershave or petrol. The pain is excruciating and focused around his right eye. The right eye reddens and tears, the right eyelid droops, and the right nostril runs. He becomes severely agitated during attacks, often pacing the room or rocking back and forth. Physical examinations, lumbar puncture, brain magnetic resonance imaging (including pituitary views), and pituitary function blood tests are normal.

A

Cluster headache

How well did you know this?
1
Not at all
2
3
4
5
Perfectly
13
Q

A 56-year-old man presents to the emergency department with headache, fever, blurred vision, and somnolence followed shortly by unresponsiveness to verbal commands. For the last 2 weeks he had been feeling ill and had decreased appetite and myalgias. Three days prior to presentation he experienced intermittent confusion, severe headache, and fever. Examination was limited by a generalised tonic-clonic seizure, for which he received lorazepam.

A

encephalitis

How well did you know this?
1
Not at all
2
3
4
5
Perfectly
14
Q

A 19-year-old man presents to the emergency department with a witnessed generalised tonic-clonic seizure episode. One month previously he had an upper respiratory tract infection. Over the last 2 weeks he developed headaches, blurred vision, generalised weakness, and progressive difficulty in walking. Examination revealed pain on eye movement as well as limb and gait ataxia.

A

encephalitis

How well did you know this?
1
Not at all
2
3
4
5
Perfectly
15
Q

A 45-year-old homeless man is found unconscious in the street. He appears stiff, with continuously shaking extremities, foaming at the mouth, and urinary incontinence. On arrival to the emergency department, he has stopped shaking but is still unconscious. Stiffening and shaking resume a few minutes later. Two empty medicine bottles are found in his pocket, labelled phenytoin and valproic acid.

A

status epilepticus

How well did you know this?
1
Not at all
2
3
4
5
Perfectly
16
Q

A 15-year-old girl wakes up disoriented and confused. She remains still in bed, looking continuously around the room as if daydreaming. When asked about her strange behaviour, she does not appear to understand and replies with unintelligible words. For the last 3 years, she has been having subtle early morning body jerks and has been told by her teachers that she frequently stares and seems inattentive in class.

A

status epilepticus

How well did you know this?
1
Not at all
2
3
4
5
Perfectly
17
Q

A 16-year-old boy presents to the emergency department with a first-time seizure event after attending an all-night party and consuming alcohol. Witnesses described the seizure as beginning abruptly with bilateral limb stiffening, followed by jerking movements in all limbs; the patient has no memory of warning symptoms prior to the seizure. The event seemed to last about 1 minute, and the patient was quite somnolent afterwards. Further review of the history reveals that the patient has been experiencing ‘jerks’ in the morning after awakening, usually involving the arms and shoulders and occasionally causing him to drop things. These ‘jerks’ do not seem to present a problem during the rest of the day.

A

generalised tonic clonic seizure

How well did you know this?
1
Not at all
2
3
4
5
Perfectly
18
Q

A 55-year-old woman recently diagnosed with a brain tumour in the left hemisphere has a witnessed seizure event. The seizure is initially recognised when the patient begins staring and is unresponsive to those around her. She seems to be picking at her clothes with her left hand, but the right arm and leg are not moving. After 20 seconds, she displays rapid head-turning and eye deviation to the right, with tonic extension of the right arm and flexion of the left arm. This is quickly followed by tonic extension of the left arm as well, then clonic jerking occurring in both arms synchronously. This jerking gradually slows and stops after about 30 seconds. The patient then becomes quite somnolent, and she appears to be using her arm and leg less on the right than the left.

A

generalised tonic clonic seizure

How well did you know this?
1
Not at all
2
3
4
5
Perfectly
19
Q

A 6-year-old female without a significant past medical history presents for evaluation of frequent unusual episodes for the past 3 months. The unusual episodes consist of sudden activity arrest with staring and minimal eyelid flutter for 10 to 20 seconds occurring 5 to 10 times per day. The patient is unresponsive to voice or tactile stimulation during the episodes. She is able to immediately resume activities without any recollection of the event once the episode finishes. Her teachers have noted that she stares off in class repeatedly and does not seem to be remembering instructions and classroom material. The diagnosis of attention-deficit/hyperactivity disorder had been suggested. One such unusual episode is induced in front of medical staff with hyperventilation.

A

absence seizure

How well did you know this?
1
Not at all
2
3
4
5
Perfectly
20
Q

An 18-year-old girl presents with several episodes of confusion over the past several months. Typically, she experiences a warning signal, which she describes as a rising sensation within her abdomen that travels upwards through her chest. She is usually unaware for a few minutes, but others have told her that she smacks her lips, picks at her clothing, and is unable to speak during these episodes. After the event she feels tired, has a headache, and prefers to lie down. She notes that her memory has not been as good as it was in the past, and her school grades have declined. Her past medical history is notable for several febrile seizures as a young child, although she was not treated for seizures at that time. An aunt was diagnosed with seizures many years ago.

A

focal seizure

How well did you know this?
1
Not at all
2
3
4
5
Perfectly
21
Q

A 70-year-old man presents with a generalised tonic-clonic seizure. His wife states that during the past month there have been times when he does not respond, mumbles words that do not make sense, and stares. After several minutes he is usually responsive. His past medical history includes hypertension and hypercholesterolaemia. He had a stroke during the preceding year. Although he recovered significantly, he still walks with a limp on the left side.

A

focal seizure

How well did you know this?
1
Not at all
2
3
4
5
Perfectly
22
Q

A 53-year-old black woman complains of a sudden, excruciating headache while sitting at work. The headache is diffuse, intense, and accompanied by nausea and vomiting. She describes the headache as the worst headache of her life. She loses consciousness following onset of the headache and is on the floor for less than 1 minute. She is being treated for hypertension and is a smoker. On examination she has a normal mental state, meningismus, bilateral subhyaloid haemorrhages, and right third cranial nerve palsy. There are no sensory deficits or weakness.

A

SAH

Brain computed tomography (CT) reveals diffuse subarachnoid blood in basal cisterns and sulci.

How well did you know this?
1
Not at all
2
3
4
5
Perfectly
23
Q

A 20-year-old woman with no significant past medical history presents with lower back pain and bilateral foot and hand tingling. Her symptoms rapidly progress over 4 days to include lower extremity weakness to the point that she is unable to mobilise her lower extremities. She reports coryzal symptoms 2 weeks ago. On examination, she has 0/5 power in her lower extremity with areflexia, but despite the paraesthesias she does not have sensory deficits. Her aminotransferases are elevated, and lumbar puncture reveals mildly elevated protein with no cells and normal glucose. She weighs 70 kg and her admission vital capacity is 1300 mL, maximum inspiratory pressure is -30 cmH₂O, and maximum expiratory pressure is 35 cmH₂O.

A

GBS

How well did you know this?
1
Not at all
2
3
4
5
Perfectly
24
Q

A 75-year-old man presents with problems walking that have developed over the previous 2 years, consisting of slow gait, imbalance (especially on turning), short stride length, and gait initiation failure. He reports urinary frequency, occasional urge incontinence, and some memory loss. On examination, his symptoms are symmetrical and much more prominent in the lower half of the body, with relative sparing of hand function, and normal facial expressiveness. He has previously been diagnosed with Parkinson’s disease; however, therapy with levodopa has not improved his symptoms.

A

hydrocephalus

How well did you know this?
1
Not at all
2
3
4
5
Perfectly
25
Q

An 18-year-old male student presents with severe headache and fever that he has had for 3 days. Examination reveals fever, photophobia, and neck stiffness.

A

meningitis - bacterial

How well did you know this?
1
Not at all
2
3
4
5
Perfectly
26
Q

A 19-year-old man presents with a 2-day history of headache and associated nausea. He says that bright light hurts his eyes. He has no significant past medical history, is not currently taking any medicine, and reports no drug allergies. He works as a librarian and has not travelled overseas for the past year. He lives with his girlfriend whom he has been seeing for 2 years. They have a pet hamster.

A

viral meningitis

How well did you know this?
1
Not at all
2
3
4
5
Perfectly
27
Q

A 40-year-old man complains of a 1-year history of twice-monthly global headache, worse on the left side in the post-auricular region. It comes on gradually and, at its most severe, the vision in his left eye becomes distorted. He often has to stop watching television as the picture becomes “blurry”. His nose becomes blocked, although sometimes he has a “runny nose”. He takes a non-steroidal anti-inflammatory drug (NSAID) that helps a little, but he feels that his head is about to explode at times. When the headache occurs, he needs to go into a dark quiet room and sleep until it resolves. He reports that the problem is “really getting him down”, and he is having difficulties with his employer due to loss of work time.

A

migraine

How well did you know this?
1
Not at all
2
3
4
5
Perfectly
28
Q

A 32-year-old woman presents with a 13-year history of 1 to 3 attacks per month of disabling pounding pain over one temple, with nausea and sensitivity to light. She says that her headaches can be triggered by lack of sleep and made worse by physical exertion, and are more common during menstrual bleeding. Untreated, they last for 2 days. On 4 occasions, headaches were preceded by the gradual appearance of a shimmering, zigzag line that enlarged, moved to the peripheral visual field, and then faded away over 45 minutes. Examination is normal.

A

migraine

How well did you know this?
1
Not at all
2
3
4
5
Perfectly
29
Q

A 60-year-old man presents with right foot drop, which has developed gradually over the last year and progressed to involve more proximal areas in the last 2 months. The patient reports associated muscle twitching and painful muscle cramps involving the same areas. The neurological examination reveals bilateral lower-extremity weakness, more severe on the right, with associated spasticity, atrophy of the right foot intrinsic muscles, diffuse fasciculations, and hyper-reflexia, with deep tendon reflexes being brisker on the right lower extremity, and a positive right Babinski’s sign. Sensation is preserved throughout. Several other family members have been diagnosed (some have died) with a pattern suggesting autosomal dominant disease

A

ALS

How well did you know this?
1
Not at all
2
3
4
5
Perfectly
30
Q

A 65-year-old woman presents with progressive slurred speech with nasal quality, and episodes of choking on liquids, for the last 4 to 5 months. Neurological examination reveals facial and tongue weakness; tongue muscle wasting and fasciculations; dysarthria; hypophonic speech; and brisk reflexes throughout (including jaw jerk).

A

ALS

How well did you know this?
1
Not at all
2
3
4
5
Perfectly
31
Q

A 28-year-old white woman who has smoked 1 pack per day for the last 10 years presents with subacute onset of cloudy vision in 1 eye, with pain on movement of that eye. She also notes difficulty with colour discrimination, particularly of reds. She was treated for a sinus infection 2 weeks ago and on further history recalls that she had a 3-week history of unilateral hemibody paraesthesias during examination week in university 6 years ago. She occasionally has some tingling on that side if she is overly tired, stressed, or hot.

A

MS

How well did you know this?
1
Not at all
2
3
4
5
Perfectly
32
Q

A 31-year-old woman with strong family history of autoimmune disease is 6 months postnatal and develops ascending numbness and weakness in both feet, slightly asymmetrically, over a period of 2 weeks. She gradually develops difficulty walking to the point where she presents to an emergency department and is also found to have a urinary tract infection.

A

MS

How well did you know this?
1
Not at all
2
3
4
5
Perfectly
33
Q

A 25-year-old woman presents with recurrent slurring of speech that worsens when she continues to talk. She has trouble swallowing, which deteriorates when she continues to eat, and has double vision that gets worse when sewing, reading, or watching TV. She reports that her head is heavy and hard to hold up. Her symptoms have progressively deteriorated over the past 6 months. She has intermittent weakness in her legs and arms. She is fearful of falling due to her legs giving way and she has trouble combing her hair or putting on deodorant. She reports a feeling of generalised fatigue and is occasionally short of breath.

A

MG

How well did you know this?
1
Not at all
2
3
4
5
Perfectly
34
Q

A 76-year-old man reports double vision for the past 2 months. Within the past 2 weeks he has developed bilateral ptosis (drooping eyelids). His ptosis is so severe at times that he holds his eyes open to read. He is unable to drive due to the ptosis and the diplopia (double vision). His symptoms are generally better in the morning and progress throughout the day.

A

MG

How well did you know this?
1
Not at all
2
3
4
5
Perfectly
35
Q

A 40-year-old man with a history of alcohol abuse is brought to the emergency department by police, who found him lying down by the side of the street. On examination he is somnolent and confused. He has a horizontal gaze palsy with impaired vestibulo-ocular reflexes and severe truncal ataxia in the presence of normal motor strength and muscle stretch reflexes.

A

Wernickes encephalopathy

How well did you know this?
1
Not at all
2
3
4
5
Perfectly
36
Q

A 30-year-old woman underwent bariatric surgery for morbid obesity. The postoperative course was complicated by a bronchopneumonia, vomiting, and poor oral intake. Four weeks after surgery she complained of vertigo and headache and soon became apathetic and developed vertical nystagmus that was worse on downward gaze.

A

Wernickes encephalopathy

How well did you know this?
1
Not at all
2
3
4
5
Perfectly
37
Q

A middle-aged woman presents with a complaint of frequent (once or twice daily for 3 weeks), brief (lasting several seconds) episodes of intense, sharp left-sided jaw pain. She has experienced these attacks for several years, but they had previously been relatively rare (1 episode daily for several consecutive days followed by months with no attacks). She says that episodes are sometimes brought on by eating but can occur without an apparent stimulus. The patient states that even though the pain is brief, she lives in fear of repeat flares.

A

trigeminal neuralgia

How well did you know this?
1
Not at all
2
3
4
5
Perfectly
38
Q

A 67-year-old man with a prior history of hypertension, diabetes, hyperlipidaemia, and a 50 pack-year smoking history noted rapid onset of right-sided weakness and subjective feeling of decreased sensation on his right side. His family reported that he seemed to have difficulty forming sentences. Symptoms were maximal within a minute and began to spontaneously abate 5 minutes later. By arrival in the emergency department 30 minutes after onset, his clinical deficits had largely resolved with the exception of a subtle weakness of his right hand. Forty minutes after presentation, all of his symptoms were completely resolved.

A

TIA

How well did you know this?
1
Not at all
2
3
4
5
Perfectly
39
Q

A 37-year-old woman presents with a 12-year history of episodic headaches. She experiences these 4 times a week, typically beginning at the end of a working day. The pain is generalised and described as similar to wearing a tight band around her head. The headaches are bothersome, but not disabling, and she denies any nausea or vomiting. She is slightly sensitive to noise but has no photophobia. Pain during her attacks typically responds to ibuprofen. Examination reveals tenderness of her scalp and both trapezius muscles.

A

tension type headache

How well did you know this?
1
Not at all
2
3
4
5
Perfectly
40
Q

A 25-year-old man presents to the emergency department after an automobile accident. He was ejected from the vehicle. He complains of numbness in both lower extremities and cannot move his legs. There is no pinprick sensation below the umbilicus except for an anal wink, and there is no rectal tone. The bulbocavernosus reflex is weakly present. Power in the lower extremities is graded at 1/5.

A

SC compression

How well did you know this?
1
Not at all
2
3
4
5
Perfectly
41
Q

A 40-year-old woman presents with back pain and difficulty with her gait. She has a long history of smoking and has had some haemoptysis recently. Her examination reveals diminished pinprick sensation from the nipple line caudally, power in the lower extremities of 4/5, absent joint position sense in the lower extremities, and diminished vibratory sense. Anal sphincter tone is intact.

A

SC compression

How well did you know this?
1
Not at all
2
3
4
5
Perfectly
42
Q

A 26-year-old woman presents with multiple sessile fleshy skin lesions. She has been aware of multiple café au lait spots since early childhood, although she ignored them as they were deemed to be birthmarks. The truncal skin lumps that led to her presentation began to appear (or become prominent) during the early second trimester of her recent pregnancy, at the end of which she delivered a female infant with multiple light brown birthmarks. Physical examination of the woman shows café au lait spots, bilateral axillary freckling, and multiple cutaneous neurofibromas over the trunk and proximal limbs. She has no neurological abnormalities. A slit-lamp ophthalmological examination reveals multiple iris Lisch nodules bilaterally. The diagnosis of NF1 is substantiated on clinical grounds. Genetic counselling clarifies the 50% recurrence risk of NF1, and respecting the patient’s wish to have additional children free of NF1, a blood sample is obtained for a molecular diagnosis in anticipation of using the data for antenatal diagnosis, or preimplantation diagnosis and selective implantation of NF1-free conceptuses.

A

NF1

How well did you know this?
1
Not at all
2
3
4
5
Perfectly
43
Q

A 69-year-old man presents with a 1-year history of mild slowness and loss of dexterity. His handwriting has become smaller, and his wife feels his face is less expressive and his voice softer. Over the last few months he has developed a subtle tremor in the right hand, noted while watching television. His symptoms developed insidiously but have mildly progressed. He has no other medical history, but he has noted some mild depression and constipation over the last 2 years. His examination demonstrates hypophonia, masked facies, decreased blink rate, micrographia, and mild right-sided bradykinesia and rigidity. An intermittent right upper extremity resting tremor is noted while he is walking. The rest of his examination and a brain MRI are normal.

A

parkinsons

How well did you know this?
1
Not at all
2
3
4
5
Perfectly
44
Q

A 42-year-old school teacher presents with difficulty managing her classroom. She has become increasingly irritable with students and fails to complete assigned tasks on time. Her sister and husband report that she has become restless, pays less attention to her appearance and social obligations, and at times is anxious and upset. She has stumbled unexpectedly. Her symptoms resemble those of her mother when she was diagnosed with Huntington’s disease. On examination, her speech is somewhat uneven and she is inappropriately flippant. Subtracting serial 7s from 100, while seated with her eyes closed, brings out random ‘piano-playing’ movements of the digits along with other movements of the limbs, torso, and face. Subtraction errors occur with this task. She is unable to keep her tongue fully protruded for 10 seconds. Finger tapping is slower than the examiner’s, and tapping tempo is uneven. Tandem walking is impaired.

A

Huntingdons

How well did you know this?
1
Not at all
2
3
4
5
Perfectly
45
Q

A 70-year-old right-handed man is discovered by a family member to have difficulty speaking and comprehending spoken language, and an inability to raise his right arm. He was last known to be fully functional 1 hour ago when the family member spoke to him by phone. There is a history of treated hypertension and diabetes.

A

Ischaemic stroke

How well did you know this?
1
Not at all
2
3
4
5
Perfectly
46
Q

A 70-year-old man with a history of chronic hypertension and atrial fibrillation is witnessed by a family member to have nausea, vomiting, and right-sided weakness as well as difficulty speaking and comprehending language. The symptoms started with only mild slurred speech before progressing over several minutes to severe aphasia and right arm paralysis. The patient is taking warfarin.

A

Haemorrhagic stroke

How well did you know this?
1
Not at all
2
3
4
5
Perfectly
47
Q

A 76-year-old white woman is brought to her general practitioner by her children because she is becoming more forgetful. She used to pay her bills independently and enjoyed cooking but has recently received overdue notices from utility companies and found it difficult to prepare a balanced meal. She has lost 3.5 kg in the past 3 months, and left the water running in her bathtub and flooded the bathroom. When her children express their concerns, she becomes irritable and resists their help. Her house has become more cluttered and unkempt. On a past visit to her physician, she had normal laboratory tests for metabolic, haematological, and thyroid function. The current evaluation reveals no depressive symptoms and 2/15 on the Geriatric Depression Scale short-form. Her Mini-Mental State Examination score is 20/30.

A

alzheimers demential

How well did you know this?
1
Not at all
2
3
4
5
Perfectly
48
Q

A 55-year-old man who worked as a technician developed difficulty finding words 2 years earlier, which has evolved into dysfluency, frequent repetition of remarks and questions, stereotypies (purposeless behaviours or fragments of speech frequently repeated, without regard to context), and echolalia (reflexive repetition of another’s speech). In the past year, he has also become forgetful. His work efficiency deteriorated due to his poor comprehension, reasoning, planning, and completion, resulting in disability leave. He also became unfeeling, intrusive (indiscriminately approaching strangers), child-like, and impulsive. He developed rigid routines (e.g., insistence on the same TV shows) and coarse manners (e.g., eating out of serving bowls, jumping queues, and walking away from conversation). Restlessness is marked: each day he bikes, swims many laps, runs 10 km, and ‘volunteers’ at a local nursing home, making the rounds with all maintenance crews. On examination, he is pleasant and cooperative. Given opportunity, he quizzes the examiner on trivial facts (such as listing capital cities). Depression is not evident, and he does not have euphoria, psychosis, or paranoia. Speech is mildly non-fluent. Verbal fluency is impaired. Mini-mental state examination score is 29.

A

FT dementia

How well did you know this?
1
Not at all
2
3
4
5
Perfectly
49
Q

A 65-year-old man presents with difficulty in decision-making and planning, which is of abrupt onset and occurs 3 months after a stroke. He has strong vascular risk factors, including smoking. Over time, there has been a fluctuating stepwise reduction in cognitive function. There is a history of nocturnal confusion and incontinence. On examination there is evidence of focal neurological deficit with pseudobulbar palsy and extrapyramidal signs. Neuroimaging indicates a probable vascular aetiology with white matter changes and infarction.

A

vascular dementia

How well did you know this?
1
Not at all
2
3
4
5
Perfectly
50
Q

A 78-year-old woman presents with confusion, agitation, and visual hallucinations. She has become progressively confused over the past 2 years and has had trouble managing her affairs, including shopping and paying bills. It is unclear when her confusion started. Initially, she was having trouble following conversations and got lost on several occasions. Her memory, which was previously good, has begun to deteriorate. At night, she sees children playing in her house and has called the police on several occasions. She gets angry easily and has been paranoid about her relatives and their intentions. Her behaviour tends to fluctuate from day to day. She started to shuffle about 6 months ago and had difficulty getting out of chairs, and getting dressed to go out seemed to take hours. On one occasion, she fell and was taken to the emergency department but was subsequently discharged with no diagnosis given.

A

Lewy body dementia

How well did you know this?
1
Not at all
2
3
4
5
Perfectly
51
Q

A 65-year-old man presents with generally decreased vision and difficulty driving at night due to glare from oncoming headlights. He describes having trouble reading the small print on his television screen. He is healthy and has no history of any other eye problems. His best corrected visual acuity is noted to be 20/50 in the right eye and 20/40 in the left eye. On examination, a yellowish opacification of the lens in the left eye is noted. On ophthalmoscopy, the red reflex in the left eye is obscured centrally, and the details of the fundus are indistinct. No other abnormalities are found.

A

cataracts

How well did you know this?
1
Not at all
2
3
4
5
Perfectly
52
Q

A 27-year-old man presents following an incident where he was struck in the left eye with a paint ball. He notices a sudden decrease in vision in the left eye, from 20/20 before the accident, to counting-fingers vision after the accident. On examination, the left pupil appears whitish, and visual acuity is greatly decreased. The patient does not have any history of other medical problems. On dilated eye examination, the lens in the left eye appears whitish anteriorly, with a spoke-like pattern. On direct ophthalmoscopy, the red reflex is diminished and retinal details are indistinct.

A

cataracts

How well did you know this?
1
Not at all
2
3
4
5
Perfectly
53
Q

A 50-year-old woman, who has no eye symptoms, is found during routine ophthalmic examination to have elevated intraocular pressure of 42 mmHg in both eyes. Funduscopy shows that the optic nerve head appears normal, with no evidence of glaucomatous neuropathy. Gonioscopy shows that the anterior chamber angles are closed for almost the full circumference.

A

Angle closure glaucoma

How well did you know this?
1
Not at all
2
3
4
5
Perfectly
54
Q

A 64-year-old woman presents to the emergency department with severe pain around her right eye of 4-hour duration, accompanied by blurred vision in that eye. She is also nauseated. Examination shows a red right eye with oedematous cornea and a wide pupil that is unresponsive to light. Intra-ocular pressure is extremely elevated (60 mmHg), only in the right eye. The anterior chamber angle is closed in both eyes.

A

Angle closure glaucoma

How well did you know this?
1
Not at all
2
3
4
5
Perfectly
55
Q

A 50-year-old man presents for a routine eye examination with no symptoms. He has elevated intra-ocular pressure of 25 mmHg in the right eye and 30 mmHg in the left eye. On dilated examination, the cup-to-disc ratio is 0.5 in the right eye and 0.8 in the left eye. Corneal thickness and gonioscopy are normal. Subsequent automated testing of visual fields demonstrates peripheral visual field loss greater in the left eye than in the right. Repeated automated visual field testing shows that the visual field defects are reproducible.

A

open angle glaucoma

How well did you know this?
1
Not at all
2
3
4
5
Perfectly
56
Q

A 40-year-old man presents to the emergency department complaining of red eye without purulent discharge. He also has pain, photophobia, blurred vision, and tearing. On slit-lamp examination, the attending ophthalmologist notices a small irregular pupil, conjunctival injection around the corneal limbus, and WBCs in the anterior chamber.

A

uveitis

How well did you know this?
1
Not at all
2
3
4
5
Perfectly
57
Q

A 30-year-old woman presents with onset of bilateral decreased vision associated with floaters. Slit-lamp examination of the anterior segment shows no abnormality. However, on dilated fundoscopic examination, vitreous cells and a choroiditis are apparent.

A

uveitis

How well did you know this?
1
Not at all
2
3
4
5
Perfectly
58
Q

A 65-year-old woman presents with a chief complaint of dizziness. She describes it as a sudden and severe spinning sensation precipitated by rolling over in bed onto her right side. Symptoms typically last <30 seconds. They have occurred nightly over the last month and occasionally during the day when she tilts her head back to look upwards. She describes no precipitating event prior to onset and no associated hearing loss, tinnitus, or other neurological symptoms. Otological and neurological examinations are normal except for the Dix-Hallpike manoeuvre, which is negative on the left but strongly positive on the right side.

A

BPPV

How well did you know this?
1
Not at all
2
3
4
5
Perfectly
59
Q

A 40-year-old woman presents with a 1-year history of recurrent episodes of vertigo. The vertigo spells are described as a sensation of the room spinning that lasts from 20 minutes to a few hours and may be associated with nausea and vomiting. The spells are incapacitating and are accompanied by dizziness, vertigo, and disequilibrium, which may last for days. No loss of consciousness is reported. The patient also reports aural fullness, tinnitus, and hearing loss in the right ear that is more pronounced around the time of her vertigo spells. Physical examination of the head and neck is normal. A horizontal nystagmus is noted. She is unable to maintain her position during Romberg’s testing or the Fukuda stepping test. She turns towards the right side and she is unable to walk tandem. Her cerebellar function tests are normal.

A

Menieres

How well did you know this?
1
Not at all
2
3
4
5
Perfectly
60
Q

A 6-year-old previously healthy boy presents with acute onset of fever of 39°C (102°F), severe throat pain that is exacerbated by swallowing, headache, and malaise. On examination his tonsils are symmetrically enlarged and red, with purulent exudate. He has multiple enlarged, painful anterior neck lymph nodes, but no other lymphadenopathy and no splenomegaly. He has no runny nose or cough, and no difficulty breathing.

A

tonsillitis

How well did you know this?
1
Not at all
2
3
4
5
Perfectly
61
Q

A 24-year-old woman presents with a 3-day history of painful sores in the genital area, dysuria, fever, and headache. She is sexually active with men and has a new partner within the past month. She does not use condoms. Physical examination reveals a temperature of 38.3°C (100.9°F), stable vital signs, slight nuchal rigidity (implying aseptic meningitis), bilateral tender inguinal lymphadenopathy, and multiple tender 1- to 2-cm erythematous ulcerations without labial crusts. The cervix is oedematous with pustules and clear discharge. Cervical motion tenderness is also present.

A

HSV inf

How well did you know this?
1
Not at all
2
3
4
5
Perfectly
62
Q

A 25-year-old man presents for STI screen. He is sexually active with men, has had four partners in the past year, and uses condoms ‘most of the time’. He was HIV-negative 6 months ago and denies a history of urethral discharge, dysuria, or genital ulcers. He does have occasional genital itching and mild sores on the penile shaft. Genitourinary examination reveals a circumcised male with no inguinal lymphadenopathy, no lesions on the penile shaft or perianal area, and no urethral discharge.

A

HSV inf

How well did you know this?
1
Not at all
2
3
4
5
Perfectly
63
Q

A 16-year-old student presents with fever, sore throat, and fatigue. She started feeling ill 1 week ago. Her symptoms are gradually getting worse, and she has difficulty swallowing. She has had a fever every day, and she could hardly get out of bed this morning. She does not remember being exposed to anybody with a similar illness recently. On physical examination she is febrile and looks ill. Enlarged cervical lymph nodes, exudative pharyngitis with soft palate petechiae and faint erythematous macular rash on the trunk and arms are found.

A

Infectious mono

How well did you know this?
1
Not at all
2
3
4
5
Perfectly
64
Q

A 42-year-old woman presents with bilateral breast pain of mild-to-moderate intensity. Pain is worse just before and improves a few days after the start of her menstrual period. Gynaecological history is significant for grava 2 para 2. She does not take an oral contraceptive. Physical examination of the breasts demonstrates diffuse nodularity throughout both breasts.

A

fibrocystic breasts

How well did you know this?
1
Not at all
2
3
4
5
Perfectly
65
Q

A 28-year-old woman in her second post-partum week presents with recent-onset breast pain and a tender wedge-shaped area in one breast that feels firm, warm, and swollen, and appears erythematous. She has decreased milk output, flu-like symptoms, pyrexia of 38°C (100.4°F), and myalgia, in addition to feeling fatigued.

A

Mastitis/breast abscess

How well did you know this?
1
Not at all
2
3
4
5
Perfectly
66
Q

A 30-year-old woman with a history of mastitis presents with sharp shooting breast pain and an exquisitely tender, swollen, red, and warm fluctuant peri-areolar breast mass.

A

Mastitis/breast abscess

How well did you know this?
1
Not at all
2
3
4
5
Perfectly
67
Q

A 65-year-old white woman presents to her doctor for a routine screening mammogram, which demonstrates a cluster of pleomorphic micro-calcifications that are located in the upper outer quadrant of her left breast. One year ago, her mammogram showed no abnormalities; the patient has been diligent in undergoing annual mammograms because her mother was diagnosed with breast cancer at the age of 50 years.

A

Breast cancer

How well did you know this?
1
Not at all
2
3
4
5
Perfectly
68
Q

A 17-year-old boy is brought to the hospital by the police owing to an overdose of unknown drugs after a fight with his girlfriend. Initial physical examination is unremarkable except for a BP of 149/99 mmHg and sinus tachycardia at a rate of 130 bpm. However, shortly after arrival in the emergency department, the patient seizes and requires intubation. Serum electrolyte panel and ABG analysis reveal a wide anion-gap metabolic acidosis. Screen for drugs of abuse and alcohol is negative. The patient is found to have a serum salicylate level of 94.8 mg/dL.

A

aspirin overdose

How well did you know this?
1
Not at all
2
3
4
5
Perfectly
69
Q

A 48-year-old woman presents to the emergency department with vague complaints of feeling unwell over the past several days. She reports generalised body pains and consequently has taken Goody’s powder, paracetamol, diphenhydramine, and several other cold preparations over the past 72 hours. Since the night before presentation, she appeared confused and provided vague responses to questions. In addition, she cannot understand that she has been ingesting multiple medications that included the same chemical. On physical examination, she is noted to have hyperpnoea. Laboratory investigations reveal a serum bicarbonate level of 9 mmol/L and a salicylate level of 50.6 mg/dL.

A

aspirin overdose

How well did you know this?
1
Not at all
2
3
4
5
Perfectly
70
Q

A 21-year-old man presents to the emergency department with central nervous system depression, respiratory depression, and miosis (1 mm pupils). Friends state that the patient was seen injecting himself at a party, at which time he became unresponsive. He is deeply unresponsive to pain and gives no history. The patient is a known drug user. He has track marks on both upper extremities and syringes are found among his belongings.

A

opiod overdose

How well did you know this?
1
Not at all
2
3
4
5
Perfectly
71
Q

A 38-year-old woman presents with nausea and pain from a tooth abscess. On further questioning she states that she has been ingesting small handfuls of paracetamol-containing pills every few hours over the previous 2 to 3 days.

A

paracetamol OD

How well did you know this?
1
Not at all
2
3
4
5
Perfectly
72
Q

A 30-year-old woman presents with 4-month history of recurrent oral and genital ulcers. She gets the oral ulcers every other week, >5 at a time, and they resolve on their own in 7 to 10 days. They cause discomfort and occur in the inner lips and cheeks and on her tongue. The genital ulcers are fewer in number and not always painful. She has also noticed acne on her legs and on her back, but not on her face, although she never had any facial acne as a teenager. She has also had 2 episodes of painful, red, round lesions on her legs. These resolved without treatment after 1 week.

A

Bechets

How well did you know this?
1
Not at all
2
3
4
5
Perfectly
73
Q

A 28-year-old man presents with a 2-month history of eye pain and blurring of vision that has been getting worse over the last several weeks. Both of his eyes are involved. He also complains of recurrent oral and genital ulcers that have been bothering him for the last 5 months. He has had facial acne for some time, but now is getting acne on his back, upper arms, and legs.

A

Bechets

How well did you know this?
1
Not at all
2
3
4
5
Perfectly
74
Q

A 14-year-old girl presents in severe respiratory distress to the emergency department. Her past medical history includes asthma and a peanut and tree nut allergy. Shortly after ingestion of a biscuit in the school cafeteria, she began complaining about flushing, pruritus, and diaphoresis followed by throat tightness, wheezing, and dyspnoea. The school nurse called an ambulance. No medications were administered and the patient did not have an epinephrine (adrenaline) auto-injector prescribed by her allergist. Her physical examination reveals audible wheezing and laryngeal oedema and an oxygen saturation of 92%.

A

anaphylaxis

How well did you know this?
1
Not at all
2
3
4
5
Perfectly
75
Q

A 65-year-old man reports being stung while working in his garden. He removed the sting and found the dying bee. In the past he tolerated insect stings on several occasions without reaction. On this occasion, within minutes, he experienced flushing, sweating, and a brief loss of consciousness. Too confused to call for help, he was found 10 minutes later by his wife. On arrival of an ambulance he was rousable, without respiratory distress or rash. Systolic BP was 75 mmHg and pulse rate was 55 bpm.

A

anaphylaxis

How well did you know this?
1
Not at all
2
3
4
5
Perfectly
76
Q

A 26-year-old female bank clerk is 24 weeks pregnant and is offered an HIV rapid test as part of her antenatal care. Her test is positive and confirmed on a second rapid test. She is referred for general HIV care. At the HIV clinic she explains that she has been very well with only pregnancy-related nausea and mild fatigue. This is her first pregnancy. On examination, she looks well, with mild generalised lymphadenopathy only. She has been married for 2 years and had only 1 sexual partner in the last 4 years. An HIV test at 20 years of age was negative. A CD4 count is performed and she is staged as WHO stage 1. She receives counselling regarding risks to her unborn child and information about prevention of mother-to-child transmission. She has not yet disclosed her status to her partner and needs assistance with this, as well as further information about positive living and initiation of antiretroviral therapy.

A

HIV

How well did you know this?
1
Not at all
2
3
4
5
Perfectly
77
Q

A 32-year-old male taxi driver was found to be HIV-infected during a recent hospitalisation for a pneumonic illness. Compatible chest x-ray findings and confirmatory sputum culture were positive for Mycobacterium tuberculosis , resulting in a diagnosis of pulmonary tuberculosis (TB). In consideration of this diagnosis, the patient had agreed to HIV testing in the hospital. HIV serology was positive by rapid HIV testing and this was confirmed on a second blood specimen. The patient was informed of the diagnosis and referred for outpatient care. In the outpatient clinic, history obtained from the patient confirmed some months of deteriorating health. He had lost approximately 10 kg in weight and had experienced fevers, night sweats, loss of appetite, and intermittent bouts of diarrhoea. In addition, 4 weeks prior to admission he had developed a productive cough and pleuritic chest pain. He had also noted a scaly skin condition at the hair line. His medical history is non-significant, but he nursed his mother with TB approximately 6 years ago. His current medicine includes anti-tuberculous therapy and pyridoxine. He has recently completed 1 week of topical mycostatin for oral candidiasis. On examination he is thin, with evidence of oral thrush and mild seborrhoeic dermatitis. He has mild bronchial breathing in his right upper chest, with mild tracheal deviation to the right. His neurological, cardiovascular, and abdominal examinations are normal. A CD4 count performed while the patient was still in the hospital was 186 cells/microlitre. He was clinically staged, based on history and findings, as World Health Organization (WHO) stage 3. A baseline viral load, full blood counts, and liver function tests are ordered prior to initiation of antiretroviral therapy. The patient discloses that he is married and has 3 children aged 6 years, 4 years, and 13 months. They are all well. Implications for testing the family for HIV are discussed with the patient.

A

HIV

How well did you know this?
1
Not at all
2
3
4
5
Perfectly
78
Q

A 42-year-old Nigerian woman presents to her primary care physician with a 2-day history of fever, chills, and sweats with associated headache and myalgia. She is febrile (38.6°C [101.4°F]) and tachycardic, but examination is otherwise unremarkable. A presumptive diagnosis of influenza is made, and she is advised to return if she does not improve. Two days later she presents to the emergency department with similar symptoms and frequent vomiting. On examination she appears ill, with a temperature of 38.8°C (101.8°F), pulse rate 120 beats per minute, blood pressure 105/60 mmHg, and mild jaundice. Further history reveals that she recently visited family in Nigeria for 2 months, returning 1 week before presentation. She did not take malaria prophylaxis.

A

Malaria

How well did you know this?
1
Not at all
2
3
4
5
Perfectly
79
Q

A 28-year-old man presents to his physician with a 5-day history of fever, chills, and rigors, not improving with paracetamol (acetaminophen), along with diarrhoea. He had been travelling in Central America for 3 months, returning 8 weeks ago. He had been bitten by mosquitoes on multiple occasions, and although he initially took malaria prophylaxis, he discontinued it due to mild nausea. He does not know the specifics of his prophylactic therapy. On examination he has a temperature of 38°C (100.4°F), and is mildly tachycardic with a blood pressure of 126/82 mmHg. The remainder of the examination is normal.

A

Malaria

How well did you know this?
1
Not at all
2
3
4
5
Perfectly
80
Q

A 35-year-old woman is admitted to hospital because of pain and swelling of the right thigh. The patient has been in excellent health until the morning before admission, when she observed a pimple on her right thigh. During the course of the day, the lesion enlarged, with increasing pain, swelling, and erythema, and was accompanied by nausea, vomiting, and delirium. Her temperature is 37.5°C (99.5°F), pulse is 128 bpm, and respirations are 20 breaths/minute. BP is 85/60 mmHg. On physical examination, the patient appears ill and in pain. A small, indurated area of skin breakdown with surrounding erythema and warmth is present on the right thigh; no fluctuance is detected. She is unable to flex or extend the right hip because of pain and reports pain on passive extension of the right ankle. The temperature soon rises to 38.4°C (101°F), and the BP drops to 70/40 mmHg. Haematocrit is 42, WBC count 5900/mm³ (with 64% neutrophils, 19% band forms), serum creatinine 168 micromol/L (1.9 mg/dL), and serum urea 7.8 millimol/L (22 mg/dL). Contrast-enhanced CT shows a diffuse, non-enhancing, honeycomb pattern within the subcutaneous tissue of the right thigh. Subcutaneous stranding and thickening of the skin are prominent in the posterolateral aspect of the thigh; there is also thickening of the posterolateral deep fascia.

A

necrotising fasciitis

How well did you know this?
1
Not at all
2
3
4
5
Perfectly
81
Q

A 10-year-old Samoan girl presents with a 2-day history of fever and sore joints. Further questioning reveals that she had a sore throat 3 weeks ago but did not seek medical help at this time. Her current illness began with fever and a sore and swollen right knee that was very painful. The following day her knee improved but her left elbow became sore and swollen. While in the waiting room her left knee is now also becoming sore and swollen.

A

rheumatic fever

How well did you know this?
1
Not at all
2
3
4
5
Perfectly
82
Q

A 31-year-old woman presents with a 1-week history of fever, chills, fatigue, and unilateral ankle pain. Her past medical history includes mitral valve prolapse and hypothyroidism. She admits to infrequent intravenous heroin use and has a 10-pack-year history of smoking. Physical examination reveals temperature of 39°C (102°F), regular heart rate 110 beats per minute, blood pressure 110/70 mmHg, and respiration rate of 16 breaths per minute. Her cardiovascular examination reveals a grade 2/4 holosystolic murmur that is loudest at the right upper sternal border. Her right ankle appears red and warm, and is very painful on dorsiflexion.

A

Infective Endocarditis

How well did you know this?
1
Not at all
2
3
4
5
Perfectly
83
Q

A 65-year-old male smoker with hypertension, dyslipidaemia, and diabetes mellitus presents with chest pain. ECG changes suggest an acute myocardial infarction. He is taken for an urgent coronary angiogram. Three days later, he is noticed to have developed an elevated serum creatinine, oliguria, and hyperkalaemia.

A

AKI

How well did you know this?
1
Not at all
2
3
4
5
Perfectly
84
Q

A 35-year-old man with a history of congenital valvular heart disease undergoes a dental procedure without appropriate antibiotic prophylaxis. Several weeks later, he presents with fever and respiratory distress. He is intubated, and Streptococcus viridans is isolated in all blood cultures drawn at the time of admission. Echocardiography demonstrates a mitral valve vegetation. Laboratory tests reveal a rising serum creatinine and urine output decline. Urine analysis reveals more than 20 white blood cells, more than 20 red blood cells, and red cell casts. Urine culture is negative. Renal ultrasound is unremarkable. Serum erythrocyte sedimentation rate is elevated.

A

AKI

How well did you know this?
1
Not at all
2
3
4
5
Perfectly
85
Q

A 60-year-old man presents to his primary care physician with a 3-month history of increasing urinary frequency without burning and nocturia 3 times each evening. He has limited his fluid consumption and caffeine intake in the evening without much benefit. There is no personal or family history of prostate cancer. Examination demonstrates no suprapubic mass or tenderness. A rectal examination demonstrates normal rectal tone and a moderately enlarged prostate without nodules or tenderness.

A

BPH

How well did you know this?
1
Not at all
2
3
4
5
Perfectly
86
Q

A 72-year-old man presents with a 6-month history of weak stream, straining, and hesitancy. There is no history of prostate cancer. The physical examination demonstrates a severely enlarged prostate without nodules. There is moderate suprapubic fullness prior to voiding. A urinalysis is normal and the prostate-specific antigen level is 3.0 micrograms/L (3.0 nanograms/mL).

A

BPH

How well did you know this?
1
Not at all
2
3
4
5
Perfectly
87
Q

A 64-year-old man presents with painless haematuria. He had a similar episode 1 year ago and was given antibiotics for a presumed urinary infection and his bleeding resolved. He has a decreased urinary stream and nocturia twice a night. He has smoked a pack of cigarettes daily for 45 years. Physical examination shows only moderate enlargement of the prostate. Urinalysis is positive for 10 to 15 RBCs and 5 to 10 WBCs per high-power field with no bacteria detected.

A

bladder cancer

How well did you know this?
1
Not at all
2
3
4
5
Perfectly
88
Q

A 54-year-old man with a 10-year history of diabetes and hypertension, with complications of diabetic retinopathy and peripheral neuropathy, presents to his primary care physician with complaints of fatigue and weight gain of 4.5 kg over the past 3 months. He denies any changes in his diet or glycaemic control, but does state that he has some intermittent nausea and anorexia. He states that he has noticed that his legs are more swollen at the end of the day but improve with elevation and rest. Physical examination reveals an obese man with a sitting blood pressure of 158/92 mmHg. The only pertinent physical examination findings are cotton wool patches and micro-aneurysms bilaterally on fundoscopic examination and pitting, bilateral lower-extremity oedema.

A

CKD

How well did you know this?
1
Not at all
2
3
4
5
Perfectly
89
Q

A 21-year-old man presents with a 3-day history of worsening left-sided scrotal pain and swelling. He reports noticing a white urethral discharge over the last 24 hours. He is otherwise fit and well, and takes no regular medicine. He is heterosexual and has a single female partner, with whom he has unprotected intercourse. Examination reveals a tender, erythematous, swollen left hemiscrotum with a palpably thickened epididymis.

A

epididymitis

How well did you know this?
1
Not at all
2
3
4
5
Perfectly
90
Q

A 74-year-old man with a known history of benign prostatic enlargement and insulin-requiring type 2 diabetes presents with a 7-day history of worsening right-sided scrotal pain and swelling. Initial symptoms of dysuria and frequency have resolved since his family doctor prescribed a course of antibiotics 4 days ago. Examination reveals a tender, swollen right epididymis with an associated hydrocele.

A

epididymitis

How well did you know this?
1
Not at all
2
3
4
5
Perfectly
91
Q

A 35-year-old man with no past medical history presents to the emergency department after he noted cola-coloured urine. He denies pain or fever associated with the bleed, but has had a sore throat for the past 3 days, which is getting better. He has not had a similar episode previously. Examination reveals a non-blanching purpuric rash over both his legs. There are no other abnormalities.

A

glomerulonephritis

How well did you know this?
1
Not at all
2
3
4
5
Perfectly
92
Q

A 42-year-old man with a medical history of HIV infection presents to his general practicioner with generalised swelling progressive for the past week. HIV was diagnosed a year ago and he has been non-compliant with the therapy prescribed. He denies orthopnoea, abdominal pain, nausea, and blood in his urine. He has non-pitting oedema mostly over the lower extremities but extending up to mid-abdomen.

A

glomerulonephritis

How well did you know this?
1
Not at all
2
3
4
5
Perfectly
93
Q

A 12-month-old boy presents to his primary care physician with a right scrotal mass. The mass is smaller in the morning than in the evening and increases significantly in size during crying. It gets smaller again when he is lying down. He has no gastrointestinal or urinary symptoms. Physical examination demonstrates normal findings on the left side of the scrotum and a non-tender soft swelling on the right side. The mass is transilluminated when a light is shone on the scrotum, suggesting it is fluid-filled. The right testicle is palpable after gentle pressure reduces the swelling.

A

hydrocele

How well did you know this?
1
Not at all
2
3
4
5
Perfectly
94
Q

A 30-year-old woman with a family history (i.e., father, aunt, and grandfather) of polycystic kidney disease (PKD) comes to the renal clinic for evaluation. She denies any history of flank pain, pyelonephritis, or haematuria, but reports having had 2 urinary tract infections (UTIs) over the last year. She is contemplating having a family in the near future. She was recently screened for this disease with an abdominal ultrasound. This showed several small echogenic foci and small cystic changes in the liver. Several bilateral kidney cysts were seen (with the largest measuring 3.2 cm), and an adjacent renal calculus. She denies any history of migraines or headaches. There is no family history of aneurysms or cerebrovascular events. She had an ambulatory blood pressure (BP) monitor study performed prior to her evaluation revealing normal BP. Her examination is completely normal.

A

PKD

How well did you know this?
1
Not at all
2
3
4
5
Perfectly
95
Q

A 40-year-old man discovered that he had PKD about 15 years ago when he had renal colic. He was found to have bilateral stones at the time and was treated with lithotripsy. A stone was analysed. He thinks it was a uric acid stone but is not sure. He has had no further renal colic or passage of stones since that time. About 10 years ago, he developed hypertension that has been treated since with adequate control, by his account. He denies having had any UTIs. He had repair of a left inguinal hernia when he was a teenager. Recently, he had a bout of gross painless haematuria lasting 3 days and went to the emergency department for evaluation. A computed tomography (CT) scan was performed, which showed no change in his polycystic kidneys compared with findings on a CT scan 1 year prior. Over the last several years, he has experienced increasing abdominal girth and has developed early satiety and dyspnoea on exertion. He denies any mechanical low back pain.

A

PKD

How well did you know this?
1
Not at all
2
3
4
5
Perfectly
96
Q

A 65-year-old white man presents to his general practitioner in his normal state of health. He describes nocturia (1 episode per night) and a 3-hour daytime voiding interval. He denies any incontinence, haematuria, dysuria, frequency, or urgency. He has no gastrointestinal complaints. Physical examination reveals his prostate to be smooth and symmetrical, with an approximate volume of 40 mL.

A

prostate cancer

How well did you know this?
1
Not at all
2
3
4
5
Perfectly
97
Q

A 60-year-old black man presents to his general practitioner with complaints of difficulty with urination. He describes a weak stream and a sense of incomplete voiding. He describes nocturia (5 episodes per night) and has been taking an alpha-blocker for this with minimal improvement. He says he can last about 60 to 90 minutes without urinating. He denies any suprapubic tenderness, dysuria, or haematuria. He further denies any back pain or gastrointestinal complaints. Rectal examination reveals his prostate to be approximately 60 mL, asymmetrical, with a large 2-cm nodule at the right base.

A

prostate cancer

How well did you know this?
1
Not at all
2
3
4
5
Perfectly
98
Q

A 68-year-old man with known coronary artery disease and peripheral vascular disease presents with recurrent episodes of flash pulmonary oedema, worsening kidney function, and progressively difficult-to-control hypertension. An angiogram of the aorta and renal arteries shows a sclerotic aorta with plaque extending into the proximal third of both renal arteries.

A

RAS

How well did you know this?
1
Not at all
2
3
4
5
Perfectly
99
Q

A 32-year-old woman with no prior medical history is seen for worsening headache and is found to have a BP of 180/110 mmHg. Her BP responds inadequately to thiazide diuretics and calcium-channel blockers. A magnetic resonance angiogram of the renal arteries reveals a beaded appearance indicative of fibromuscular dysplasia.

A

RAS

How well did you know this?
1
Not at all
2
3
4
5
Perfectly
100
Q

A 56-year-old obese woman presents to the emergency department with a history suggestive of biliary colic, including epigastric discomfort after a heavy meal. Her past medical history includes cholelithiasis, hypertension (treated with an angiotensin-converting enzyme [ACE] inhibitor), and dyslipidaemia (treated with a statin). She is an ex-smoker, drinks alcohol socially, and has no significant family history. On palpation of her abdomen, she has RUQ pain, but there are no other relevant findings on examination. An abdominal ultrasound is performed, which demonstrates the presence of gallbladder stones without obstruction, and an incidental 5-cm, left-sided renal mass.

A

Renal cell carcinoma

How well did you know this?
1
Not at all
2
3
4
5
Perfectly
101
Q

A thin 65-year-old man with no significant past medical history presents with a 5-month history of right-sided flank discomfort and abdominal fullness. He finally seeks medical attention because of 2 weeks of lower extremity oedema, and 4 days of gross haematuria with clots. On examination, his blood pressure is 160/90 mmHg, heart rate is 120 bpm and regular, and he is afebrile. He is found to have a palpable right-sided lower abdominal mass, and pitting oedema to the mid-shins bilaterally, which is worse on the right.

A

Renal cell carcinoma

How well did you know this?
1
Not at all
2
3
4
5
Perfectly
102
Q

A 35-year-old man presents with non-specific testicular discomfort and the feeling of a mass in the testis. On examination, a 2 cm by 1 cm smooth, painless mass is palpated in the right testis. The mass does not transilluminate with light. There is no lymphadenopathy.

A

testicular cancer

How well did you know this?
1
Not at all
2
3
4
5
Perfectly
103
Q

A 13-year-old boy developed sudden-onset unilateral scrotal pain that woke him from sleep. He presents with left scrotal pain, nausea and vomiting, and left lower abdominal pain. On examination, he has a tender, enlarged, high-riding left testicle with a transverse lie. There is an absent cremasteric reflex on the left.

A

torsion

How well did you know this?
1
Not at all
2
3
4
5
Perfectly
104
Q

A 27-year-old, healthy, sexually active woman presents with pain on urination and recent onset of urinary frequency and urgency. She has no costovertebral angle tenderness on examination.

A

UTI

How well did you know this?
1
Not at all
2
3
4
5
Perfectly
105
Q

A 59-year-old man complains of urinary frequency, urgency, and dysuria for several days. He denies the presence of haematuria or penile discharge, but does have 3 episodes of nocturia most nights. His past medical history includes benign prostatic hyperplasia (BPH). The patient is in a monogamous relationship with his wife.

A

UTI

How well did you know this?
1
Not at all
2
3
4
5
Perfectly
106
Q

A 45-year-old man presents to the emergency department with a 1-hour history of sudden onset of left-sided flank pain radiating down towards his groin. The patient is writhing in pain, which is unrelieved by position. He also complains of nausea and vomiting.

A

renal stone

How well did you know this?
1
Not at all
2
3
4
5
Perfectly
107
Q

A 15-year-old boy presents with left scrotal swelling/mass detected on a routine school physical examination. The patient states that he is completely asymptomatic. There is no significant medical history and he has not had any previous surgeries. He is on no medicines and has no allergies. Physical examination in the supine position reveals asymmetrical testicular size (left smaller than right) with no masses. With the patient in the standing position, a grade III left varicocele can clearly be seen and palpated in the left hemiscrotum.

A

varicocele

How well did you know this?
1
Not at all
2
3
4
5
Perfectly
108
Q

A 30-year-old healthy man presents with primary infertility. He has been unable to establish a pregnancy for the last 12 months with his partner. On physical examination, a grade II left varicocele is easily palpable when the patient is standing and is non-palpable when supine. The testicles are symmetrical and normal in size.

A

varicocele

How well did you know this?
1
Not at all
2
3
4
5
Perfectly
109
Q

A 47-year-old man presents with arthritic pain of knees and hips, soft-tissue swelling, and excessive sweating. He also noticed progressive enlargement of the hands and feet. He has been taking antihypertensive medicine for the past 3 years. On physical examination, he has coarse facial features with prognathism and prominent supra-orbital ridges. The tongue is enlarged and the fingers are thickened. His wife complains that he frequently snores. Laboratory work-up reveals an elevated plasma insulin-like growth factor 1 (IGF-1) concentration of 73 nanomols/L (560 micrograms/L or 560 nanograms/mL) (normal for age, 16 to 31 nanomols/L [120 to 235 micrograms/L or 120 to 235 nanograms/mL]) and a basal plasma growth hormone level of 15 micrograms/L (15 nanograms/mL). MRI examination of the sella turcica region shows a 14 mm pituitary mass with right cavernous sinus invasion.

A

acromegaly

How well did you know this?
1
Not at all
2
3
4
5
Perfectly
110
Q

A 48-year-old man has a 4-month history of increasing fatigue and anorexia. He has lost 5.5 kg and noticed increased skin pigmentation. He has been otherwise healthy. His mother has Hashimoto’s thyroiditis and one of his sisters has type 1 diabetes. His blood pressure is 110/85 mmHg (supine) and 92/60 mmHg (sitting). His face shows signs of wasting and his skin has diffuse hyperpigmentation, which is more pronounced in the oral mucosa, palmar creases, and knuckles.

A

Addisons

How well did you know this?
1
Not at all
2
3
4
5
Perfectly
111
Q

A 54-year-old woman with hypothyroidism complains of persistent fatigue, despite adequate thyroxine replacement. She has noticed increasing lack of energy for the past 3 months and additional symptoms of anorexia and dizziness. She also has significant loss of axillary and pubic hair. Her blood pressure is 105/80 mmHg (supine) and 85/70 mmHg (sitting). The only abnormal finding on physical examination is a mild increase in thyroid size, with the thyroid having rubbery consistency.

A

Addisons

How well did you know this?
1
Not at all
2
3
4
5
Perfectly
112
Q

A 12-year-old white girl is brought to the emergency department by her parents due to 12 hours of rapidly worsening nausea, vomiting, abdominal pain, and lethargy. Over the last week she has felt excessively thirsty and has been urinating a lot. Physical examination reveals a lean, dehydrated girl with deep rapid respirations, tachycardia, and no response to verbal commands.

A

T1D

How well did you know this?
1
Not at all
2
3
4
5
Perfectly
113
Q

An overweight 55-year-old woman presents for preventative care. She notes that her mother died of diabetes, but reports no polyuria, polydipsia, or weight loss. BP is 144/92 mmHg, fasting blood sugar 8.2 mmol/L (148 mg/dL) (on 2 occasions), HbA1c 65 mmol/mol (8.1%), LDL-cholesterol 5.18 mmol/L (200 mg/dL), HDL-cholesterol 0.8 mmol/L (30 mg/dL), and triglycerides 6.53 mmol/L (252 mg/dL).

A

T2D

How well did you know this?
1
Not at all
2
3
4
5
Perfectly
114
Q

A 60-year-old man presents with a 3-year history of diarrhoea, with no clear precipitating factors. Over the past few months he has noticed flushing affecting his face. These episodes occur at any time but are worse during times of stress and exercise. His wife has also noticed intermittent reddening of his face, which lasts for a few minutes. More recently he has not tolerated alcohol, chocolate, or bananas.

A

carcinoid

How well did you know this?
1
Not at all
2
3
4
5
Perfectly
115
Q

A 50-year-old woman presents with a long history of atypical flushing, initially attributed to menopause. The flushing is associated with purplish discolouration of the face with each episode lasting 30 minutes. She also reports palpitations on exertion and recurrent episodes of abdominal pain.

A

Carcinoid

How well did you know this?
1
Not at all
2
3
4
5
Perfectly
116
Q

At a routine examination, a 65-year-old woman is discovered to have hypercalcaemia. Follow-up laboratory tests show synchronously elevated serum calcium and intact parathyroid hormone, with low phosphorus and mildly elevated alkaline phosphatase. 25-hydroxyvitamin D is in the low normal range. Past medical history is significant for hypertension and coronary artery disease. Review of symptoms includes complaints of fatigue, feeling achy, and vague depression and mental fatigue. The patient has a history of nephrolithiasis and newly detected osteopenia. Family history is negative for renal stones or calcium disorders.

A

primary hyperpara

How well did you know this?
1
Not at all
2
3
4
5
Perfectly
117
Q

A 34-year-old woman presents with complaints of weight gain and irregular menses for the last several years. She has gained 20 kg over the past 3 years and feels that most of the weight gain is in her abdomen and face. She notes bruising without significant trauma, difficulty rising from a chair, and proximal muscle wasting. She was diagnosed with type 2 diabetes and hypertension 1 year ago.

A

Cushing

How well did you know this?
1
Not at all
2
3
4
5
Perfectly
118
Q

A 54-year-old man presents for evaluation of an incidentally discovered adrenal nodule. He underwent a CT scan of the abdomen for evaluation of abdominal pain, which was negative except for a 2 cm well-circumscribed, low-density (2 Hounsfield units) nodule in the right adrenal gland. He reports weight gain of 15 kg over the past 4 years. He has difficult-to-control type 2 diabetes and hypertension. He has had 2 episodes of renal colic in the last 5 years.

A

Cushings

How well did you know this?
1
Not at all
2
3
4
5
Perfectly
119
Q

A 42-year-old man undergoes trans-sphenoidal surgery for a large, non-functioning pituitary macro-adenoma. Preoperatively, dynamic pituitary hormone tests were normal, as was his fluid intake and output. Two days following surgery he developed acute polyuria, extreme thirst, and polydipsia. His urine output over the next 24 hours was 6 litres, with frequent nocturia.

A

DI

How well did you know this?
1
Not at all
2
3
4
5
Perfectly
120
Q

A 75-year-old woman presents to her family physician with a 6-month history of progressive fatigue and malaise with polyuria, polydipsia, and nocturia. She has a long-standing history of bipolar affective disorder, and has been receiving lithium for the past 15 years.

A

DI

How well did you know this?
1
Not at all
2
3
4
5
Perfectly
121
Q

A 20-year-old man is brought to the accident and emergency department with abdominal pain, nausea, and vomiting with increasing polyuria, polydipsia, and drowsiness since the previous day. He was diagnosed with type 1 diabetes 2 years previously. He mentions that he ran out of insulin 2 days ago. Vital signs at admission are: BP 106/67 mmHg, heart rate 123 beats per minute, respiratory rate 32 breaths per minute, temperature 37.1°C (98.8°F). On mental status examination, he is drowsy. Physical examination reveals Kussmaul breathing (deep and rapid respiration due to ketoacidosis) with acetone odour and mild generalised abdominal tenderness without guarding and rebound tenderness. Initial laboratory data are: blood glucose 25.0 mmol/L (450 mg/dL), arterial pH 7.24, pCO2 25 mmHg, bicarbonate 12 mmol/L (12 mEq/L), WBC count 18.5 × 10^9/L (18,500/microlitre), sodium 128 mmol/L (128 mEq/L), potassium 5.2 mmol/L (5.2 mEq/L), chloride 97 mmol/L (97 mEq/L), serum urea 11.4 mmol/L (32 mg/dL), creatinine 150.3 micromol/L (1.7 mg/dL), serum ketones strongly positive.

A

DKA

How well did you know this?
1
Not at all
2
3
4
5
Perfectly
122
Q

A 72-year-old man is brought to hospital from a nursing home for progressive lethargy. The patient has a history of hypertension complicated by a stroke 3 years previously. This has impaired his speech and rendered him wheelchair-bound. He also has a schizothymic disorder for which he was started recently on clozapine. On presentation, he is disoriented to time and place and febrile, with a temperature of 38.3°C (101°F). Vital signs include a BP of 106/67 mmHg, heart rate of 106 beats per minute, and a respiratory rate of 32 breaths per minute. Initial laboratory work-up reveals a serum glucose of 52.7 mmol/L (950 mg/dL), a serum sodium of 127 mmol/L (127 mEq/L), a serum urea of 21.1 mmol/L (59 mg/dL), and a serum creatinine of 175.4 micromol/L (2.3 mg/dL). Serum osmolality is calculated as 338 mmol/kg (338 mOsm/kg). Urinalysis reveals numerous white blood cells and bacteria. Urine is positive for nitrates but negative for ketones. Serum is negative for beta-hydroxybutyrate.

A

HHS

How well did you know this?
1
Not at all
2
3
4
5
Perfectly
123
Q

A 43-year-old pilot presents for a stress test required by his employer. He states that there is a strong history of premature cardiac disease in his family and 2 of his older brothers are currently being treated for high cholesterol. System review is negative except for some mild shortness of breath with exercise. Examination demonstrates moderate abdominal obesity with a body mass index of 31 kg/m² and waist circumference of 102 cm (40 inches). The remainder of the examination is normal.

A

hypercholesterolaemia

How well did you know this?
1
Not at all
2
3
4
5
Perfectly
124
Q

A 63-year-old woman with diabetes presents with an episode of retrosternal chest pain and diaphoresis that occurred while walking up stairs earlier that day. Her examination is unremarkable except for blood pressure 156/96 mmHg and abdominal obesity. A recent lipid profile showed triglyceride level 3.8 mmol/L (335 mg/dL), total cholesterol 6.29 mmol/L (243 mg/dL), low-density lipoprotein cholesterol 3.678 mmol/L (142 mg/dL), and high-density lipoprotein cholesterol 0.88 mmol/L (34 mg/dL). Her electrocardiogram shows no acute changes.

A

hypertriglyceridaemia

How well did you know this?
1
Not at all
2
3
4
5
Perfectly
125
Q

A 27-year-old man presents for evaluation of infertility. He had a normal birth and early development, but did not undergo puberty. He has developed none of the typical male secondary sexual characteristics. He reports diminished libido, although he occasionally gets early morning erections. He is able to get an erection during sexual activity, provided he is sufficiently aroused. He married 3 years ago, but attempts to father a child have been unsuccessful. His wife has normal menstrual cycles and her gynaecological examination is normal. On examination, the patient has sparse facial hair and low hair line. Minimal bilateral, non-tender gynaecomastia is present. The patient has a normal penis and scrotum, but the testes are small and firm with an estimated volume of 4 mL each.

A

hypogonadism

126
Q

A 42-year-old man presents with a 4-month history of frontal headaches and decreased libido. The symptoms started gradually. In addition to decreased libido, the patient also complains of gradual loss of early morning erections. He has been married for 12 years and has 3 children. He denies any head or testicular injury. He also denies gynaecomastia or galactorrhoea. The only abnormality on his physical examination is bitemporal hemianopia on visual field testing.

A

hypogonadism

127
Q

A 45-year-old white woman presents with symptoms of fatigue, depression, and mild weight gain. Physical examination demonstrates heart rate of 58 beats per minute, coarse dry skin, and bi-lateral eyelid oedema. Serum TSH is 40 milli-international units/L (mIU/L) (normal range, subject to laboratory standards, 0.35-6.20 mIU/L), and free T4 is 6.44 picomol/L (0.5 nanograms/dL) (usual normal range, subject to laboratory standards, 9.00-23.12 picomol/L [0.8 to 1.8 nanograms/dL]). Therapy is begun with levothyroxine 100 micrograms daily and the patient’s symptoms improve. Repeat testing 6 weeks later reveals a normal TSH (5 mIU/L). The patient is maintained on this dose and repeat TSH testing is planned yearly or if symptoms recur.

A

hypothyroidism

128
Q

A 25-year-old woman presenting with renal colic also complains of new-onset headaches, fatigue, and constipation. Her menstrual cycle is regular and she has not experienced episodes of flushing. Her weight is unchanged and her peripheral vision is normal. Family history reveals that her father had kidney stones and died of a ‘stomach problem’ in his 60s. Examination is unremarkable with visual fields full to confrontation.

A

MEN

129
Q

An 18-year-old man with no medical history presents with a lump on his neck that he noticed while shaving. A 2-cm thyroid nodule is palpable. There is nothing else of note on examination.

A

MEN

130
Q

A 72-year-old man is evaluated for increasing fatigue and bone pain. His medical history is significant for chronic alcoholism, lactose intolerance, and a vertebral compression fracture 1 year ago. He is housebound without any sunlight exposure. He denies any personal or family history of kidney stones, fractures, or osteoporosis. His physical examination is remarkable for generalised tenderness of the long bones and proximal muscle weakness, with difficulty climbing stairs and a waddling gait.

A

osteomalacia

131
Q

A 70-year-old woman presents to the emergency department after falling while getting out of bed. She sustained an intertrochanteric fracture of the right hip. Preoperative chest x-ray before repair of the hip reveals that she had existing asymptomatic vertebral fractures before her fall.

A

osteoporosis

132
Q

A late middle-aged woman presents with chronic right hip and anterior thigh pain, with increased localised temperature. Lately she has needed a walking stick. During the past 6 months her relatives have noticed a progressive hearing loss on her left side, as well as some facial changes - mostly enlargement of her mandible.

A

Pagets

133
Q

A 33-year-old woman presents to her doctor complaining of a several-month history of episodic palpitations and diaphoresis. She states that her husband noticed that she becomes pale during these episodes. She has been experiencing progressive episodic headaches, which are not relieved by paracetamol. In the past, she has been told that she had a high calcium level. She has a history of kidney stones. Her family history is unremarkable; specifically, there is no history for tumours, endocrinopathies, or hypertension. Physical examination reveals a BP of 220/120 mmHg and hypertensive retinal changes.

A

Phaeo

134
Q

A 50-year-old obese woman with long-standing, poorly controlled diabetes presents with lethargy and fatigue. Screening labs report that she has a creatinine level of 190.6 micromol/L (2.5 mg/dL) and a blood urea nitrogen level of 14.3 nanomol/L (40 mg/dL). Additional labs are ordered, which reveal a calcium level of 1.85 mmol/L (7.4 mg/dL) and a phosphorus level of 1.9 mmol/L (5.9 mg/dL). The parathyroid hormone level is 400 nanograms/L (400 picograms/mL).

A

secondary hyperpara

135
Q

An 85-year-old female nursing-home patient is being seen for post-menopausal skeletal disease that has become a concern after she fell and broke her wrist. Her bone densitometry reveals osteoporosis (T-score: -3.5). Lab tests return with a calcium level of 2.2 mmol/L (8.8 mg/dL) and a parathyroid hormone level of 120 nanograms/L (120 picograms/mL). These results prompt vitamin D testing that returns a 25-hydroxyvitamin D level of 14 nanograms/mL.

A

secondary hyperpara

136
Q

An 18-year-old woman presents with a chief complaint of hirsutism. She needs to wax her upper lip and chin twice a week. This has been a problem for 4 years. She also has excess hairs on her upper back and lower abdomen. Her periods are irregular, occurring every 2 to 3 months. Embarrassment about the facial hirsutism has affected her social life, and she is finding she feels depressed much of the time.

A

PCOS

137
Q

A 32-year-old women presents with a chief complaint of difficulty becoming pregnant. She was prescribed oral contraceptives at the age of 17 years because of irregular periods (4 to 6 periods per year). She continued with oral contraception until 30 years of age, at which point she and her husband decided they wanted to have a baby. Since ceasing oral contraception, she has gained weight and has only 3 to 5 periods per year. She has actively been trying to conceive, with no results.

A

PCOS

138
Q

A 54-year-old man presents with a 10-year history of hypertension that has been difficult to control with antihypertensive medicines. His symptoms include frequent headaches, nocturia (3-4 times per night), and lethargy. He has no other medical conditions or past medical history. Apart from a BP of 160/96 mmHg, findings on physical examination are unremarkable. Plasma electrolytes are normal.

A

Conns

139
Q

A 45-year-old man presents with loss of libido and some erectile dysfunction. He is otherwise healthy. On physical examination he has mild bilateral gynaecomastia and normal testes. Laboratory work-up reveals a highly elevated prolactin level of 46,000 mIU/L (2300 micrograms/L). Normal prolactin levels are up to 300 mIU/L (15 micrograms/L). He also has low testosterone, LH, and FSH levels. MRI examination of the pituitary sella depicts a large 32 mm pituitary macro-adenoma with suprasellar extension and optic chiasmal compression. Visual field assessment reveals bi-temporal hemianopia.

A

Prolactinoma

140
Q

A 27-year-old woman presents with amenorrhoea. She had been taking the combined oral contraceptive pill for the last 9 years, stopping this 11 months ago. She is otherwise healthy, but on physical examination she has bilateral galactorrhoea. Laboratory work-up reveals an elevated prolactin level of 3000 mIU/L (150 micrograms/L). Normal prolactin levels are up to 500 mIU/L (25 micrograms/L). She also had low-normal gonadotrophin (luteinising hormone [LH], follicle-stimulating hormone [FSH]) levels. Magnetic resonance imaging (MRI) examination of the pituitary sellar region depicts a 6 mm right-sided pituitary mass, with no suprasellar or parasellar extension.

A

Prolactinoma

141
Q

A 76-year-old homeless white man presents to the emergency department after police find him disoriented on the streets in late August. The patient gives little history, but admits to ongoing cough with productive sputum, night sweats/chills, and mild dyspnoea. He proceeds to suffer from a seizure. Vital signs demonstrate an elevated temperature at 38.7°C (101.7°F), a respiration rate of 26 breaths per minute, 94% oxygen saturation (on 3 L of O2), and pulse 87 bpm, with no evidence of orthostatic hypotension. Physical examination demonstrates a malnourished and dishevelled man in a postictal state. There is no sign of injury to the body. Crackles can be heard at the right lung base. Lab work demonstrates serum sodium of 120 mmol/L (120 mEq/L), serum creatinine of 88 micromol/L (1.0 mg/dL), and negative alcohol and toxicology screens. CXR demonstrates a large infiltrate in the right lower lung, consistent with pulmonary infection or abscess.

A

SIADH

142
Q

A 40-year-old woman is found to have a 2-cm right-sided thyroid nodule during a routine physical examination. She has no history of head and neck irradiation or family history of thyroid cancer. The nodule is firm and mobile in relation to the underlying tissue. Vital signs and the remainder of the examination are normal.

A

thyroid cancer

143
Q

A 35-year-old woman has a history of 3 consecutive pregnancy losses before 12 weeks of pregnancy. She had no other known complications during the pregnancies. Further testing reveals a lupus anticoagulant, which is still present on repeat testing 12 weeks later. Physical examination is normal.

A

antiphospholipid syndrome

144
Q

A 42-year-old man is referred because of central retinal vein thrombosis. Medical history is uneventful; in particular, he has no known risk factors for venous or arterial thromboembolic disease. Screening for antiphospholipid antibodies reveals moderately elevated anticardiolipin antibody levels on 2 occasions, 12 weeks apart.

A

antiphospholipid syndrome

145
Q

A 30-year-old man presents with fever and sore throat of 2 days’ duration. He reports several months of increasing fatigue and exertional dyspnoea, as well as easy bruising. Examination reveals tachycardia, evidence of tonsillopharyngitis, and scattered ecchymoses.

A

aplastic anaemia

146
Q

A 45-year-old man with acute onset of pancreatitis presents with episodes of epistaxis, increased PT/PTT, and decreased platelet count. Further coagulation work-up reveals increased thrombin time, decreased fibrinogen level, positive D-dimers, and increased fibrin degradation products. The blood culture is negative.

A

DIC

147
Q

A 20-year-old black woman presents to her primary care physician complaining of generalised weakness, fevers, and light-headedness for 2 weeks. Her symptoms have worsened over the previous week, when she developed left lower chest pain and left upper quadrant abdominal pain. A urinalysis was obtained, and she was treated for a UTI. She returns to her primary care physician when symptoms continue to worsen.

A

haemolytic anaemia

148
Q

An 18-month-old boy presents with left ankle swelling and pain. He has limited range of motion at the ankle and has difficulty walking. Over the last year, he has presented with significant haematomas at immunisation sites. He also had prolonged bleeding after heel prick for neonatal screening tests.

A

haemophilia

149
Q

A 4-year-old boy presents with a 7-day history of abdominal pain and watery diarrhoea that became bloody after the first day. Three days before the onset of symptoms, he had visited a fairground with his family and had eaten a burger. Physical examination reveals a mild anaemia.

A

HUS

150
Q

A 50-year-old woman presents approximately 3 weeks after an upper respiratory tract illness with petechiae, easy bruising, and gum bleeding. She has no personal or family history of a bleeding disorder and takes no medicines. Physical examination is normal except for petechiae and bruising. Specifically, she has no lymphadenopathy or hepatosplenomegaly. Full blood count reveals thrombocytopenia with a platelet count of 12 × 10⁹/L (12 × 10³/microlitre) but other cell lines are within normal limits. Peripheral blood smear shows thrombocytopenia but no other abnormalities.

A

ITP

diag of exc

151
Q

A 38-year-old man presents to his primary care physician complaining of generalised weakness, epistaxis, mouth ulcers, and weight loss. He has unremarkable past medical history and takes no medications. Physical examination reveals mild pallor and petechial haemorrhages over his lower limbs. He has multiple, widespread small lymph nodes that are palpable, and mild splenomegaly.

A

ALL

152
Q

A 4-year-old girl presents with lethargy, dyspnoea, fever, and bruising. On examination she has hepatosplenomegaly. Chest x-ray shows a mediastinal mass and pleural effusion.

A

ALL

153
Q

62-year-old man presents to his primary care physician for an annual physical. He denies any complaints such as fever or chills, weight loss, or fatigue. Of note, his blood tests show an elevated WBC count. The WBCs are predominantly lymphocytes, with a differential of 80% lymphocytes and an absolute lymphocyte count of 75 x 10⁹/L (75 x 10³/microlitre).

A

CLL

154
Q

A 60-year-old man presents with swollen lymph nodes in the cervical and inguinal region that have been present for 2 months and are gradually increasing in size. The lymphadenopathy is painless and has not responded to a course of antibiotics prescribed by the primary care physician. The patient denies any recent history of infection, fever, or chills. A blood test shows an elevated WBC count. The WBCs are predominantly lymphocytes, with a differential of 88% lymphocytes and an absolute lymphocyte count of 80 x 10⁹/L (80 x 10³/microlitre).

A

CLL

155
Q

A 58-year-old man presents to his primary care physician with increasing tiredness, accompanied by bruising on his legs. He also complains of aching bones. He has no previous illnesses. On examination, he is pyrexial and pale, has bony tenderness over the sternum and tibia, and has petechiae on his legs. There are no palpable lymph nodes. He has crepitations at the left base. The liver and spleen are not palpable.

A

AML

156
Q

A 54-year-old man presents to his primary care physician with a 2-month history of fever, malaise, and weight loss. He also reports frequent epistaxis, abdominal fullness, and early satiety. On examination, he is found to have splenomegaly.

A

CML

157
Q

A 50-year-old man presents to his primary care physician for a routine physical examination. He is asymptomatic at the time of the visit and the physical examination is normal. Routine baseline bloods showed elevated WBC and platelet counts.

A

CML

158
Q

A 25-year-old male presents to his general practitioner with a slowly enlarging, non-painful right neck mass. He denies recent upper respiratory tract infections, fevers, night sweats, or unintentional weight loss. He is otherwise healthy. Social history and family history are unremarkable. On examination he is afebrile with normal vital signs. Pertinent findings include a 3-cm, firm, round, non-tender, mobile mass in the mid-right neck. There is no other peripheral lymphadenopathy. The liver and spleen are not enlarged.

A

Hodgkins lymphoma

159
Q

A 55-year-old male farmer presents with worsening shortness of breath, night sweats, fevers, bilateral axillary lymphadenopathy, and a 7.7 kg (12%) total body weight loss over 3 months. Recently, he has not been able to work because of fatigue. Physical examination revealed a 3.5 cm left axillary mass, enlarged cervical, axillary, and inguinal lymph nodes, splenomegaly, and no hepatomegaly.

A

NHL

160
Q

A 56-year-old woman presents with a painless right neck lump that has been slowly enlarging for the last 2 years. She denies fevers, night sweats, or weight loss. Physical examination reveals bilateral cervical and axillary adenopathy and a palpable spleen.

A

NHL

161
Q

A 60-year-old previously healthy man presents with 2 to 3 months of back pain. Over the last 3 weeks, he has developed a cough and increasing fatigue. On examination he has evidence of pneumonia and is noted on radiography to have osteolytic lesions. Laboratory data reveals anaemia associated with the presence of a monoclonal protein.

A

MM

162
Q

A 45-year-old woman presents to the emergency department with nausea, vomiting, and confusion. She has a history of low back pain of 6 months’ duration and increasing sciatic pain in the last 2 weeks. On physical examination, the patient is pale and dehydrated with bone tenderness in the lumbar region. Neurological examination reveals an upgoing plantar reflex on the left with intact power in all muscle groups and at all joints. Magnetic resonance imaging reveals an L5 compression fracture. This is associated with hypercalcaemia and renal insufficiency.

A

MM

163
Q

A 70-year-old man presents with generalised fatigue that has slowly progressed over several months. On physical examination, the patient has pale mucus membranes and mild tachycardia. The remainder of the examination is unremarkable.

A

myelodysplasia -most asymptomatic

164
Q

A 67-year-old man, with a long-standing history of smoking (>100 pack-years), coronary artery disease, chronic renal insufficiency, hypertension, and chronic obstructive pulmonary disease, in his routine follow-up appointment is found to have a haematocrit of 19%. He had a haematocrit of 36% the previous year. He reports fatigue, night sweats, 4.5 kg (10 lb) weight loss, abdominal discomfort, and progressive dyspnoea on exertion. He denies fever, chest pain, or upper or lower gastrointestinal bleeding. On examination, he is cachectic but not in acute distress. His conjunctivae are pale. He has mild-to-moderate hearing loss. There is no lymphadenopathy. Chest examination reveals distant heart sounds with bilateral expiratory wheezes. Cardiac examination reveals no murmurs. Abdominal examination reveals a moderately enlarged spleen without hepatomegaly. There is no peripheral oedema or clubbing. Laboratory results reveal a white blood cell count of 6.2 x 10^9/L (6200/microlitre) with an absolute neutrophil count of 2.2 x 10^9/L (2200/microlitre), an absolute lymphocyte count of 2.4 x 10^9/L (2400/microlitre), and 0.36 x 10^9/L monocytes (360 monocytes/microlitre); a haemoglobin (Hb) of 60 g/L (6 g/dL) with a mean corpuscular volume of 86; a platelet count of 96 x 10^9/L (96 x 10^3/microlitre) and a reticulocyte count of 0.6%, with an absolute reticulocyte count of 14.1 x 10^9/L (14.1 x 10^3/microlitre). The peripheral blood smear shows occasional teardrop-shaped red blood cells, and erythroblasts and myelocytes. Lactate dehydrogenase is 245 U/L. Iron studies, serum B12, and red blood cell folate levels, as well as a serum and urine protein electrophoresis, are within normal limits. Computed tomographic scans of the chest, abdomen, and pelvis reveal moderate mediastinal lymphadenopathy and splenomegaly. Upper gastrointestinal endoscopy and colonoscopy are normal. A bone marrow aspirate is a ‘dry tap’. Bone marrow biopsy reveals an entirely fibrotic marrow with a few scattered plasma cells, lymphocytes, and maturing myeloid cells. Megakaryocytes are scattered and sometimes clustered and atypical with large hyperchromic nuclei. Haemoglobin staining reveals a few erythrocyte precursors. A CD34 stain reveals no increase in blasts. Mutation testing is positive for MPL, but negative for JAK2 V617F and calreticulin.

A

Myelofibrosis

165
Q

A 72-year-old woman with a past medical history of depression, hyperlipidaemia, and basal cell carcinoma of the skin, on low-dose selective serotonin reuptake inhibitor and statin, presents with thrombocythaemia. Full blood count shows a platelet count of 650 x 10^9/L (650 x 10^3/microlitre); white blood cell count of 13.2 x 10^9/L (13200/microlitre) with an absolute neutrophil count of 7.7 x 10^9/L (7700/microlitre), an absolute lymphocyte count of 4.6 x 10^9/L (4600/microlitre), and monocyte count of 0.9 x 10^9/L (900/microliter); and haemoglobin of 152 g/L (15.2 g/dL). Spleen size is 11 cm on magnetic resonance imaging examination. Total blood volume and red blood cell mass are normal. Bone marrow biopsy reveals an increased number of megakaryocytes and moderate increase of reticulin fibres. Iron staining is normal and chromosomal analysis shows no abnormalities. Philadelphia chromosome is negative. Mutation testing is positive for JAK2 V617F, but negative for calreticulin and MPL.

A

Myelofibrosis

166
Q

A 55-year-old man has had routine physical examinations for several years and has always been healthy, does not smoke, and has no history of pulmonary disease. His primary care physician has noted a gradually increasing haemoglobin level over the past few years (to a current level of 195 g/L [19.5 g/dL]), mild leukocytosis, and mild thrombocytosis. He has frequent episodes of facial flushing that are associated with slight headaches and a feeling of fullness in his head and neck. He has noted intermittent burning, stinging, and tingling sensations in his fingertips. He has recurrent, often severe, pruritus that is exacerbated by taking a hot bath. On examination, he has a red face and neck and the spleen is mildly enlarged.

A

PV

167
Q

A 62-year-old man, who has always been healthy, arrives for a pre-operative check prior to a minor procedure. A routine full blood count reveals an elevated haemoglobin level of 190 g/L (19.0 g/dL). He is surprised to hear about this abnormal result, as he has not noticed any symptoms or signs that have caused him concern. On examination, the only abnormality is a red facial complexion.

A

PV

168
Q

A 24-year-old woman with known sickle cell disease presents with a 3-day history of cough productive of white sputum, nausea, and poor appetite. She also has chest and hip pain unalleviated by paracetamol or ibuprofen.

A

SCD

169
Q

A 6-month-old boy with no previous medical problems presents with fever and painful swelling of the hands and feet. His parents are concerned because he has been inconsolable for 6 hours. The infant has been refusing bottles and has needed fewer nappy changes over the last 2 days.

A

SCD

170
Q

An 8-month-old boy of Mediterranean origin presents with pallor and abdominal distension, both of which are progressive. The perinatal history was uneventful, and the boy is noted to be pale, with poor feeding, decreased activity, and failure to thrive. Hepatosplenomegaly and mild bony abnormalities of the skull are noted (frontal and parietal bossing).

A

thalassemia

171
Q

A 40-year-old overweight black woman presents with a 1- to 2-week prodrome of fatigue and malaise with diarrhoea and vomiting. Examination is normal except for slight confusion and petechiae on her lower extremities. Laboratory studies show a haematocrit of 25% and a platelet count of 10 x 10⁹/L (10,000/microlitre). Lactate dehydrogenase is raised. Serum creatinine is 97.2 micromol/L (1.1 mg/dL). Peripheral smear shows fragmented red blood cells (schistocytes) and an raised reticulocyte count.

A

TTP

172
Q

A 68-year-old man presents for a routine physical examination and follow-up for his hypertension, hyperlipidaemia, and hypothyroidism. He complains of mild fatigue but is otherwise healthy. Laboratory evaluation is remarkable for a haematocrit of 0.34 (34%), with an MCV of 110 fL (110 micrometres^3). On further query, he denies alcohol use and any other symptoms.

A

vit b12 def

173
Q

A 70-year-old man presents for routine physical examination. He complains of fatigue, shortness of breath, and painful swallowing. He admits to daily alcohol consumption and decreased consumption of fresh vegetables and fruits. Physical examination reveals pallor, glossitis, flow murmur, and normal neurological examination.

A

folate def

174
Q

A 24-year-old woman presents to the emergency department 8 weeks postnatal with heavy vaginal bleeding, fatigue, and light-headedness. This was her first pregnancy. She has a history of menorrhagia since menarche and iron-deficiency anaemia. She had no bleeding symptoms during her pregnancy, and her vaginal bleeding was not excessive in the first few days after delivery, but it has continued since the delivery and in the past week has increased in flow. Her past medical history is remarkable for an appendectomy at age 14 years without bleeding complications, but she had to return to the oral surgeon for suturing after wisdom tooth extraction at age 16 years. Her family history is remarkable for a sister with heavy menses. Her father had recurrent nosebleeds as a child and had several cauterisations as therapy.

A

von willebrand

175
Q

A 60-year-old man presents with acute onset of shortness of breath, fever, and cough. A chest x-ray shows a right lower lobe infiltrate, and sputum has gram-positive diplococci. He is given intravenous antibiotics but his respiratory status declines over 24 hours. He becomes hypotensive and is transferred to the intensive care unit. He is intubated for hypoxaemia and requires vasopressors for septic shock despite adequate volume resuscitation. He requires high levels of inspired oxygen (FiO₂) and positive end-expiratory pressure (PEEP) on the ventilator to keep his oxygen saturation >90%. Repeat chest x-ray shows bilateral alveolar infiltrates, and his PaO₂/FiO₂ ratio is 109.

A

ARDS

176
Q

A 52-year-old man presents with a 6-month history of heartburn and atypical chest pain, both unrelated to food. He also described ‘gurgling’ sounds in his chest. A month before presentation he developed intermittent dysphagia to both solids and liquids, regurgitation, and weight loss of 3 kg.

A

Achalasia

177
Q

A 65-year-old woman presents to the emergency department with a 2-day history of progressive right upper quadrant (RUQ) pain that she rates as 9/10. She reports experiencing fever, and being unable to eat or drink due to nausea and abdominal pain at baseline, exacerbated by food ingestion. Her bowel movements are less frequent and have become loose (but with no diarrhoea) with bright red blood, or black and tarry. Her pain is not relieved by bowel movement and is not related to food. She has not recently taken antibiotics, nor does she use non-steroidal anti-inflammatory drugs or drink alcohol. On examination, she is febrile at 39.4°C (102.9°F); supine BP is 97/58 mmHg; standing BP is 76/41 mmHg; HR is 127 bpm; and respiratory rate is 24 breaths per minute with normal oxygen saturation. Her examination is remarkable for scleral and sublingual icterus, tachycardia, RUQ pain with no rebound, and involuntary guarding on the right side. Faecal occult blood test is negative. Laboratory results show a WBC of 18.0 × 10⁹/L (18,000/microlitre) (reference range 4.8-10.8 × 10⁹/L or 4800-10,800/microlitre) with 17% (reference range 0% to 4%) bands and PMNs of 82% (reference range 35% to 70%). AST is 207 units/L (reference range 8-34 units/L), ALT is 196 units/L (reference range 7-35 units/L), alkaline phosphatase is 478 units/L (reference range 25-100 units/L), total bilirubin is 107.7 micromol/L (6.3 mg/dL) (reference range 3.4 to 22.2 micromol/L or 0.2 to 1.3 mg/dL), and amylase is 82 units/L (53-123 units/L).

A

ascending cholangitis

178
Q

A 58-year-old man with pancreatic adenocarcinoma, who had a plastic stent placed in his common bile duct 6 weeks ago to relieve obstructive jaundice, presents to the emergency department with a 1-week history of progressive nausea and occasional vomiting after eating. He has generalised abdominal pain that is worse in the RUQ. He has experienced subjective fever/chills and states that his bowel movements are pale. Laboratory results show a WBC of 14.0 × 10⁹/L (14,000/microlitre) (reference range 4.8-10.8 × 10⁹/L or 4800-10,800/microlitre) with 8% (reference range 0% to 4%) bands and PMNs of 77% (reference range 35% to 70%). AST is 214 units/L (reference range 8-34 units/L), ALT is 181 units/L (reference range 7-35 units/L), alkaline phosphatase is 543 units/L (reference range 25-100 units/L), total bilirubin is 183.0 micromol/L (10.7 mg/dL) (reference range 3.4 to 22.2 micromol/L or 0.2 to 1.3 mg/dL), and amylase is 110 units/L (reference range 53-123 units/L).

A

ascending cholangitis

179
Q

A 45-year-old man presents to the emergency department with restlessness and tremors. He is anxious, pacing in the hallway. Initial vital signs show a heart rate of 121 beats per minute and blood pressure of 169/104 mmHg; other vital signs are normal. On further questioning by the nurse he states that he is nauseous and wants something to help with the shakes. During physician interview, the patient admits to heavy alcohol use and that he is trying to cut down on drinking. He also says that his current symptoms started to develop about 5 hours after his last drink.

A

alcohol withdrawal

180
Q

A 50-year-old man presents to his general practitioner with complaints of fatigue for 2 months. The patient also notes distension of his abdomen and shortness of breath beginning 2 weeks ago. His wife reports that the patient has been having episodes of confusion lately. The patient has a significant medical history of chronic heavy alcohol consumption of about half a pint of vodka daily for around 20 years. On physical exam the patient is noted to have scleral icterus, tremors of both hands, and spider angiomata on the chest. There is abdominal distension with presence of shifting dullness, fluid waves, and splenomegaly. Laboratory examination shows low haemoglobin, low platelets, low sodium, AST elevation > ALT elevation, and high PT and INR. Ultrasound of the abdomen shows liver hyperechogenicity, portal hypertension, splenomegaly, and ascites.

A

alcoholic hepatitis

181
Q

A 38-year-old man presents to the emergency department for severe alcohol abuse with nausea and vomiting. He has a significant medical history of chronic heavy alcohol consumption of half a pint of vodka daily for about 5 years until 1 year ago; since then he has had severe intermittent binge alcohol intake. He reports no other significant medical problems. The patient is confused and slightly obtunded, and hepatomegaly is discovered on physical exam. His BMI is 22. Pertinent positive laboratory values show low haemoglobin, AST elevation > ALT elevation, normal PT and INR, and very high serum alcohol level. Ultrasound of the abdomen shows fatty infiltration in the liver.

A

alcoholic hepatitis

182
Q

A 79-year-old man presents with dyspnoea on exertion for 1 year and lower extremity oedema. As part of a cardiac work-up, the echo shows concentric left ventricular hypertrophy. Cardiac catheterisation shows normal coronary arteries and he is referred for further evaluation of non-cardiac dyspnoea.

A

amyloidosis

183
Q

A 62-year-old man is referred for management of atypical multiple myeloma. He has a mild anaemia of 120 g/L (12 g/dL), a urinary protein loss of 2.2 g/day with a urinary immunofixation showing free lambda light chains. However, the bone marrow shows only 5% plasma cells and does not fulfil criteria for multiple myeloma.

A

amyloidosis

184
Q

A 28-year-old woman presents with a history of severe pain on defecation for the last 3 months. She has noticed a small amount of blood on the stool. The pain is severe and she is worried about the pain she will experience with the next bowel action.

A

Anal fissure

185
Q

A 22-year-old male presents to the emergency department with abdominal pain, anorexia, nausea, and low-grade fever. Pain started in the mid-abdominal region 6 hours ago and is now in the right lower quadrant of the abdomen. The pain was steady in nature and aggravated by coughing. Physical examination reveals a low-grade fever (38°C; 100.5°F), pain on palpation at right lower quadrant (McBurney’s sign), and leukocytosis (12 x 10^9/L or 12,000/microlitre) with 85% neutrophils.

A

Appendicitis

186
Q

A 12-year-old girl presents with sudden-onset severe generalised abdominal pain associated with nausea, vomiting, and diarrhoea. On examination she appears unwell and has a temperature of 40°C (104°F). Her abdomen is tense with generalised tenderness and guarding. No bowel sounds are present.

A

Appendicitis

187
Q

A 45-year-old woman presents with insidious onset of fatigue, malaise, lethargy, anorexia, nausea, abdominal discomfort, mild pruritus, and arthralgia involving the small joints. Her past medical history includes coeliac disease. Physical examination reveals hepatomegaly and spider angiomata.

A

Autoimmune hepatitis

188
Q

A 55-year-old obese man presents with frequent heartburn. He describes a post-prandial, retrosternal burning sensation following fatty and spicy meals. This symptom also frequently wakes him from sleep, with occasional coughing and a sour taste in his throat. He has tried many non-prescription antacids, which only relieve symptoms in the short term. He has suffered from this symptom for over 10 years. He denies dysphagia, odynophagia, or weight loss, but reports frequent hoarseness in the mornings. His past medical history is significant only for hypertension. His family history is unremarkable. He did smoke cigarettes, but stopped 5 years ago.

A

Barrett oesophagus

189
Q

A 65-year-old woman presents to her primary care physician with a 4-month history of intermittent abdominal pain localised to the RUQ with radiation to the epigastrium; the pain increases with the ingestion of fatty food and decreases with fasting. In the last 2 weeks the pain has been more frequent and steady. The patient complains of nausea, pruritus, anorexia, and weight loss, which she relates to the lack of appetite. At physical examination, there is RUQ tenderness and jaundice of the conjunctival sclera. No lymphadenopathy or palpable masses are found.

A

cholangiocarcinoma

190
Q

A 20-year-old obese woman with a 2-year history of gallstones presents to the emergency department with severe, constant RUQ pain, nausea, and vomiting after eating fried chicken for dinner. She denies any chest pain or diarrhoea. Three months ago she developed intermittent, sharp RUQ pains. On physical examination she has a temperature of 38°C (100.4°F), moderate RUQ tenderness on palpation, but no evidence of jaundice.

A

cholecystitis

191
Q

A 56-year-old man with a remote history of intravenous drug use presents to an initial visit complaining of increased abdominal girth but denies jaundice. He drinks about 2 to 4 glasses of wine with dinner and recalls having had abnormal liver enzymes in the past. Physical examination reveals telangiectasias, a palpable firm liver, mild splenomegaly, and shifting dullness consistent with the presence of ascites. Liver function is found to be deranged with elevated aminotransferases (AST: 90 U/L, ALT: 87 U/L), and the patient is positive for anti-hepatitis C antibody.

A

cirrhosis

192
Q

A 60-year-old woman with a past medical history of obesity, diabetes, and dyslipidaemia is noted to have abnormal liver enzymes with elevated aminotransferases (ALT: 68 U/L, AST: 82 U/L), and normal alkaline phosphatase and bilirubin. She denies significant alcohol consumption, and tests for viral hepatitis and autoimmune markers are negative. An abdominal ultrasound reveals evidence of fatty infiltration of the liver and slight enlargement of the spleen.

A

cirrhosis

193
Q

A 46-year-old woman presents with fatigue and is found to have iron deficiency with anaemia. She has experienced intermittent episodes of mild diarrhoea for many years, previously diagnosed as irritable bowel syndrome and lactose intolerance. She has no current significant gastrointestinal symptoms such as diarrhoea, bloating, or abdominal pain. Examination reveals two oral aphthous ulcers and pallor. Abdominal examination is normal and results of faecal testing for occult blood are negative.

A

coeliac

194
Q

A 9-year-old boy presents with vomiting for 5 days. His sister, who has coeliac disease, has had similar symptoms. His growth has been normal and he has not experienced any other possible symptoms of coeliac disease, except for intermittent constipation. Immunoglobulin A-tissue transglutaminase titre is 5 times the upper limit of normal.

A

coeliac

195
Q

A 70-year-old man presents to his primary care physician with a complaint of rectal bleeding. He describes blood mixed in with the stool, which is associated with a change in his normal bowel habit such that he is going more frequently than normal. He has also experienced some crampy left-sided abdominal pain and weight loss. He has previously been fit and well and there was no family history of GI disease. Examination of his abdomen and digital rectal examination are normal.

A

colorectal cancer

196
Q

A 25-year-old white man presents to his general practitioner with cramping abdominal pain for 2 days. He reports having loose stools and losing 6.8 kg over a 3-month duration. He also reports increased fatigue. On physical examination, his temperature is 37.6°C (99.6°F). Other vital signs are within normal limits. Abdomen is soft with normal bowel sounds and moderate tenderness in the right lower quadrant, without guarding or rigidity. Rectal examination is normal and the stool is guaiac positive. The rest of the examination is unremarkable.

A

crohns

197
Q

A 16-year-old girl presents to emergency care with perianal pain and discharge. She reports a 2-year history of intermittent bloody diarrhoea with nocturnal symptoms. On examination, she is apyrexial with normal vital signs. Her abdomen is soft and slightly tender on palpation in the left lower quadrant. Rectal examination is difficult to perform due to pain, but an area of erythematous swelling is visible close to the anal margin, discharging watery pus from its apex. Several anal tags are also presen

A

crohns

198
Q

A 57-year-old female with history of hypertension and hypercholesterolaemia presents to the emergency department with a 24-hour history of gradually worsening left-lower quadrant abdominal pain associated with nausea and vomiting. Prior to this episode, the patient did not have any significant gastrointestinal (GI) problems, except slight constipation and occasional dyspepsia after heavy meals. She felt feverish but did not take her temperature. Her family history is negative for GI disorders.

A

diverticular disease

199
Q

A 34-year-old mother of three presents to her family physician with a 3-week history of abdominal cramping pain in both lower quadrants. She has been having frequent small, soft stools accompanied by some mucus but no blood. Her symptoms are improved with bowel movement or passage of flatus. She has had these symptoms almost monthly since she was in college, but they have been worse recently. Past history is negative except for three normal pregnancies. Family history is negative for colon cancer. A sister has similar symptoms but has not seen a physician. Personal/social history reveals that she is an accountant working long hours. Her firm is about to merge with another, and she fears her job situation is tenuous. Review of systems is otherwise negative. She has not lost any weight or had any other constitutional symptoms. On physical examination, the only finding is some mild tenderness in the right lower quadrant. No mass is felt.

A

IBS

200
Q

A 40-year-old housewife complains of recurrent constipation. She has had problems since her 20s, but they are worse now. The constipation is accompanied by abdominal bloating and abdominal pain, and the discomfort is only better when she has a bowel movement. On her gynaecologist’s advice, she has tried more fibre in her diet, including fresh fruits and leafy vegetables, but that has only made the bloating worse. Her past history includes a cholecystectomy and a hysterectomy. Physical examination is entirely normal. Rectal examination reveals normal consistency stool. Stool samples test negative for occult blood.

A

IBS

201
Q

A 46-year-old obese woman presents with a 6-hour history of moderate steady pain in the RUQ that began after eating dinner and radiates through to her back. This pain gradually increased and became constant over the last few hours. She has had previous episodes of similar pain for which she has not sought medical advice. Her vital signs are normal. The pertinent findings on physical examination are tenderness to palpation in the right upper quadrant without guarding or rebound.

A

biliary colic
Biliary colic: is characterised by steady, severe (intensity >5 on a scale of 1 to 10) pain in the right upper quadrant of the abdomen lasting more than 15 to 30 minutes. An attack of simple biliary colic commonly requires an analgesic but should resolve within 5 hours.

Cholecystitis: biliary pain lasting more than 5 hours is accompanied by features of inflammation: fever, marked RUQ tenderness (Murphy’s sign), and leukocytosis. Some patients progress to sepsis. Occasionally, stones can perforate the gallbladder leading to intestinal obstruction (gallstone ileus).

Choledocholithiasis: when stones obstruct the bile ducts, biliary-type pain is accompanied by cholestasis which manifests as jaundice. More sinister is acute cholangitis, characterised by Charcot’s triad of biliary pain, jaundice, and fever. Acute cholangitis represents a medical emergency.

Acute pancreatitis: epigastric pain radiating to the back results from bile duct stones obstructing the pancreatic ducts. Inflammatory features include peritonitis.

202
Q

A 77-year-old man presents to his general practitioner with weight loss of 6.8 kg (15 lbs) and a 3-month history of dysphagia and abdominal pain. The only abnormal finding on physical examination is stools positive for occult blood. He is referred for an upper endoscopy, which shows an exophytic, ulcerated mass in the cardia of the stomach. Biopsy reveals moderately differentiated adenocarcinoma.

A

gastric cancer
Weight loss and abdominal pain are two of the most common presenting symptoms in patients with gastric cancer. [3] Dysphagia is more common in patients who present with proximal or gastro-oesophageal junction tumours. In patients with advanced stomach cancer, physical examination may show a left supraclavicular node (Virchow’s node), periumbilical nodule (Sister Mary Joseph’s nodule), or left axillary node (Irish node). Metastatic disease to the ovaries can present with ovarian masses (Krukenberg’s tumour) in women.

203
Q

A 42-year-old woman has heartburn after meals and a sour taste in her mouth. For the past 4 to 6 months she has had symptoms several times per week. Symptoms are worse when she lies down or bends over. Antacids help somewhat. The patient has no dysphagia, vomiting, abdominal pain, exertional symptoms, melaena, or weight loss. Past medical history and family history are non-contributory. The patient drinks alcohol occasionally and does not smoke. On physical examination, height is 1.63 m (5 feet 4 inches), weight 77.1 kg, and BP 140/88 mmHg. The remainder of the examination is unremarkable.

A

GORD

204
Q

A 40-year-old man presents to his primary care physician with a 2-month history of intermittent upper abdominal pain. He describes the pain as a dull, gnawing ache. The pain sometimes wakes him at night, is relieved by food and drinking milk, and is helped partially by ranitidine. He had a similar but milder episode about 5 years ago, which was treated with omeprazole. Physical examination reveals a fit, apparently healthy man in no distress. The only abnormal finding is mild epigastric tenderness on palpation of the abdomen.

A

PUD

205
Q

A 58-year-old white woman of North European descent presents with a 2-month history of increasing fatigue, difficulty with ambulation, and memory deficits. Family history is notable for autoimmune disease. Laboratory findings are remarkable for a macrocytic anaemia, a markedly reduced serum vitamin B₁₂, and presence of anti-parietal cell antibodies.

A

gastritis

206
Q

A 42-year-old man presents with a recent history of abdominal pain, distension, and nausea. Urea breath testing for Helicobacter pylori is positive.

A

gastritis

207
Q

A 48-year-old man presents to hospital after several episodes of vomiting blood following periods of forceful retching and vomiting. He had been binge drinking alcohol over the preceding 2 days.

A

MWT

208
Q

A 64-year-old man presents to hospital after 4 episodes of vomiting over the past 2 days. He describes the appearance of the vomit as resembling coffee grounds. Black, tarry stool was seen during rectal examination; however, no other physical findings were seen.

A

MWT

209
Q

A 70-year-old man who smokes heavily presents with a 6-month history of intermittent abdominal pain and nausea. He has lost 10 kg of weight in the past 2 months, which he thinks is due to a decreased appetite, and he complains of pruritus. On physical examination there is icterus in the conjunctival sclerae and epigastric tenderness but no abdominal mass or lymphadenopathy. Blood tests demonstrate elevated bilirubin and alkaline phosphatase; the rest of the blood tests are within the normal range.

A

Pancreatic cancer

210
Q

A 45-year-old woman presents to her physician with vague upper abdominal (epigastric) pain. After treatment with proton-pump inhibitors, analgesics, and antacids over a period of 3 months, which were ineffective, the patient also started to experience back pain. This prompted an initial upper gastrointestinal endoscopy, which was normal. Nearly 4 months after initial presentation, an upper abdominal ultrasound reveals a pancreatic mass with liver metastases.

A

pancreatic cancer

211
Q

A 53-year-old man presents to the emergency department complaining of severe mid-epigastric abdominal pain that radiates to the back. The pain improves when the patient leans forwards or assumes the fetal position and worsens with deep inspiration and movement. He also complains of nausea, vomiting, and anorexia, and gives a history of heavy alcoholic intake this past week. He is tachycardic, tachypnoeic, and febrile with hypotension. He is slightly agitated and confused. He is diaphoretic with decreased breath sounds over the base of the left lung.

A

acute pancreatitis

212
Q

A 47-year-old overweight woman is admitted with generalised abdominal pain. She has been unable to eat or drink due to nausea and vomiting. She states the pain started in the right upper quadrant, similar to previous episodes that she had been to the emergency department with over the past few months. An ultrasound obtained on her last visit to the emergency department revealed gallstones with no inflammation of the gallbladder, and she was told that she should see a surgeon. She looks jaundiced and in distress. She has point tenderness under her ribs on the right, which is worsened with deep palpation. No mass is palpable.

A

acute pancreatitis

213
Q

A 55-year-old woman presents for evaluation of a chronic cough, productive of thick, yellow sputum that sometimes becomes blood-tinged. She has experienced recurrent episodes of fever associated with pleuritic chest pain. She states that she is embarrassed by the persistent, intractable nature of her cough and has been prescribed multiple courses of antibiotics. Over the last 5 years, she has developed shortness of breath with exertion. Her past medical history is significant for pneumonia as a child and sinus polyps during adulthood for which she has had surgery.

A

bronchiectasis

214
Q

A 65-year-old man presents to his local aneurysm surveillance team for a screening ultrasound scan. He has been feeling well and in his usual state of good health. His medical history is notable for mild hypertension and he has a 100-pack-year tobacco history.

A

AAA

215
Q

A 55-year-old man with a history of hypertension (well controlled with medication) and tobacco use presents to his primary care physician with a 2-day history of constant and gnawing hypogastric pain. The pain has been steadily worsening in intensity. He says the pain radiates to his lower back and both groins at times. While he cannot identify any aggravating factors (such as movement), he feels the pain improves with his knees flexed. There is a palpable pulsatile mass just left of midline below the umbilicus. He is immediately referred for definitive management, but during transfer becomes hypotensive and unresponsive.

A

AAA

216
Q

A 59-year-old man presents to the emergency department with a sudden onset of excruciating chest pain, which he describes as tearing. There is a history of hypertension. On physical examination, his heart rate is 95 beats per minute. Blood pressure is 195/90 mmHg in the right arm and 160/80 mmHg in the left arm. Pulses are absent in the right leg and diminished in the left.

A

aortic dissection

217
Q

A 55-year-old white man presents with weakness, palpitations, and dyspnoea on exertion. On physical examination, his blood pressure is 148/50 mmHg with a bounding pulse and an early diastolic murmur over the left sternal border. He denies any history of drug abuse, rheumatic fever, or connective tissue disorder. The patient is taking hydrochlorothiazide for high blood pressure. Echocardiography reveals a left ventricular ejection fraction (LVEF) of 55%, left ventricular end-diastolic diameter of 70 mm, and end-systolic diameter of 50 mm.

A

AR

218
Q

A 31-year-old black man presents to clinic for the first time for a routine physical examination. He denies any complaints. On physical examination the only abnormality is a systolic murmur best heard over the second right intercostal space and an early diastolic murmur best heard over the third left sternal border. LVEF is 55% to 60% with mild LVH. Left ventricular end-systolic diameter is 45 mm and aortic root diameter is 3.5 cm.

A

AR

219
Q

A 50-year-old male diabetic smoker presents complaining of leg pain with exertion for 6 months. He notices that he has bilateral calf cramping with walking. He states that it is worse on his right calf than his left and that it goes away when he stops walking. He has noticed that he is able to walk less and less before the onset of symptoms.

A

PAD

220
Q

A 75-year-old woman with hypertension and hyperlipidaemia presents with abnormal ankle brachial index on a routine screening. She is able to walk without any discomfort and is active.

A

PAD

221
Q

A 65-year-old man with a history of hypertension, diabetes mellitus, and hyperlipidaemia presents to the accident and emergency department with the first episode of rapid palpitations, shortness of breath, and discomfort in his chest. These symptoms started acutely and have been present for 4 hours. Physical examination shows an irregularly irregular radial pulse at rate between 90 and 110 bpm, BP 110/70 mmHg, and respiratory rate of 20 breaths per minute. Heart sounds are irregular, but no S3 or S4 gallop or murmurs are audible. There are no other abnormalities on examination.

A

AF

222
Q

A 56-year-old woman with a 6- week history of weight loss, anxiety, and insomnia presents with palpitation and dyspnoea. Her pulse rate is irregular at 140 to 150 bpm. Her BP is 95/55 mmHg. She looks thin, frail, and rather anxious and jittery. Her palms are sweaty and have fine tremors. She has a palpable smooth goitre. Examination of the eyes shows bilateral exophthalmoses.

A

AF

223
Q

A 70-year-old woman with a history of hypertension, diabetes mellitus, hyperlipidaemia, and prior myocardial infarction presents to the emergency department with palpitations and shortness of breath. These symptoms started 2 days ago. She was diagnosed to have AF with rapid ventricular rate response a year and a half ago, at which time an attempted direct current cardioversion and a trial of sotalol to maintain sinus rhythm and prevent further episodes of AF were unsuccessful. The patient was treated with digoxin and metoprolol to control rate and warfarin to prevent stroke. Current physical examination shows that she is febrile and has an irregularly irregular radial pulse at a rate between 90 and 110 bpm, BP 100/70 mmHg, and respiratory rate of 26 breaths per minute. Heart sounds are irregular, but no S3 or S4 gallop or murmurs are audible. The breath sounds are of bronchial character associated with crepitations over left basal lung area.

A

AF

224
Q

A 78-year-old man presents to his primary care physician complaining of 2 months of progressive shortness of breath on exertion. He first recognises having to catch his breath while gardening and is now unable to walk up the stairs in his house without stopping. Previously he was healthy and active without similar complaints. On physical examination there is a loud systolic murmur at the right upper sternal border radiating to the carotid vessels.

A

AS
heaving apex beat
AS is a progressive disease that presents after a long subclinical period with symptoms of chest pain, syncope, and heart failure. While the most common complaint is dyspnoea with exertion, patients also frequently note syncope or chest pain that may be identical to that caused by CAD. Many cases of AS are diagnosed during the subclinical phase while a murmur noted on physical examination is being investigated. Even with severe AS, patients may be truly asymptomatic. A careful history is important to determine if the patient has altered his or her habits in response to slowly worsening stenosis.

225
Q

A 67-year-old woman presents to her primary care physician complaining of increasing shortness of breath, especially when trying to sleep. She has a history of hypertension and hyperlipidaemia, and is being treated with a beta-blocker and statin therapy. She does not smoke and drinks alcohol in moderation. On examination, her blood pressure is 148/83 mmHg and heart rate is 126 beats per minute. There is an audible S4 and the jugular venous pressure is elevated 3 cm above normal.

A

CHF chronic

226
Q

A 60-year-old man presents to the accident and emergency department. He reports being progressively short of breath. He has a history of uncontrolled hypertension, non-insulin-dependent diabetes mellitus, and has been a heavy smoker for more than 40 years. He underwent a successful primary angioplasty for a large acute anterior myocardial infarction 2 months ago. His blood pressure is 75/40 mmHg, his heart rate 110 beats per minute, and his respiratory rate 30. He has elevated neck veins and a prominent S3. His ECG shows sinus tachycardia, and a transthoracic echocardiogram performed in the A&E department reveals impaired systolic function, with an ejection fraction of 20%.

A

chronic CHF

227
Q

A 70-year-old woman complains of increasing exertional dyspnoea for the last 2 days and now has dyspnoea at rest. She has a history of hypertension for the last 5 years and a 35 pack-year smoking history, but no other established illnesses. Current medications are hydrochlorothiazide daily for the last 3 years. She has been prescribed lisinopril but failed to fill the prescription. On examination her BP is 190/90 mmHg, heart rate 104 bpm. There is an audible S4 and the jugular venous pressure is elevated 2 cm above normal. Lung examination reveals fine bibasal crepitations. There is no ankle oedema. Echocardiogram shows an ejection fraction of 60%.

A

acute heart failure

228
Q

A 73-year-old woman with previous history of myocardial infarction presents to the emergency department. She is breathless and finding it difficult to talk in full sentences. On examination she is centrally cyanosed with cool extremities. Her pulse is 110 bpm and systolic BP only just recordable at 80 mmHg. Jugular venous pressure is elevated 3 cm above normal and the cardiac apex beat is displaced. Respiratory rate is increased and she has widespread crackles and wheezes on chest examination. Echocardiogram shows an ejection fraction of 35%

A

acute heart failure

229
Q

A 43-year-old man with no significant medical history presents with 3 days of progressive fatigue, dyspnoea on exertion and while lying in the supine position, and lower-extremity swelling. He reports having a flu-like illness consisting of fevers, myalgias, fatigue, and respiratory symptoms 2 weeks prior that resolved spontaneously. On examination the patient has an elevated jugular venous pressure, bilateral pulmonary rales, and a heart rate of 104 bpm with an audible left ventricular S3 gallop. He is mildly dyspnoeic at rest but becomes markedly dyspnoeic with minimal exertion.

A

myocarditis

230
Q

A 49-year-old man originally from Argentina with a 3-year history of congestive heart failure presents to the emergency department with syncope while at work. He reports speaking with a co-worker then suddenly awaking on the floor of the office. The patient’s wife states that the patient has had 2 similar episodes in the past. The patient is euvolaemic with non-distended neck veins and a normal lung examination. Cardiac examination reveals a laterally displaced apex, and regular rate and rhythm without murmur or gallop but frequent ectopy.

A

myocarditis
The clinical manifestation of myocarditis is highly variable and ranges from asymptomatic ECG abnormalities to cardiogenic shock. [11] [12] When cardiac involvement manifests clinically, it typically occurs 7 to 10 days after a systemic illness. Chest pain occurs in 35% of patients and may be typical, atypical, or positional in nature. [4] Occasionally, patients present with ischaemic-sounding chest pain and ST-segment elevations on ECG that mimic acute coronary syndrome. Left ventricular dysfunction tends to be global instead of regional, and coronary angiography is normal. [13] Rarely, patients with myocarditis present with sudden cardiac death, usually due to ventricular arrhythmias.

231
Q

An otherwise healthy 30-year-old man presents with a several-day history of progressive, severe, retrosternal chest pain that is sharp and pleuritic in nature. The pain is worse on lying down and improved with sitting forward. There is radiation to the neck and shoulders and specifically to the trapezius muscle ridges. The pain is constant and unrelated to exertion. On physical examination, a pericardial friction rub is heard at end-expiration with the patient leaning forward.

A

pericarditis

232
Q

A 65-year-old woman presents with unilateral leg pain and swelling of 5 days’ duration. There is a history of hypertension, congestive heart failure, and recent hospitalisation for pneumonia. She had been recuperating at home but on beginning to mobilise and walk, the right leg became painful, tender, and swollen. On examination, the right calf is 4 cm greater in circumference than the left when measured 10 cm below the tibial tuberosity. Superficial veins on the right foot are more dilated than on the left and easily visible. The right leg is slightly redder than the left. There is tenderness on palpation in the popliteal fossa behind the right knee.

A

DVT

233
Q

A 60-year-old man with a history of diabetes, hypercholesterolaemia, and heavy smoking for over 20 years presents giving a 3-week history of increasing pain in his left forefoot, which is affecting his ability to walk and is disrupting his sleep. On examination, his left foot is pale, cold, devoid of hair, and his lateral two toes are dusky and discoloured. No foot pulses are palpable and are only just detectable by Doppler probe.

A

gangrene

234
Q

A 56-year-old man with history of poorly controlled diabetes mellitus and alcoholism presents with severe scrotal pain and fever for 3 days. He denies perianal tenderness. His vital signs are blood pressure 125/60 mmHg, heart rate 120 beats per minute, respiratory rate 25 breaths per minute, and temperature is 38.6°C (101.5°F). His scrotum is extremely tender, black, and malodorous. The adjacent perineal and femoral skin is crepitant.

A

gangrene

235
Q

A 64-year-old black man presents for a check-up. He denies past medical problems, but has been told that his blood pressure was a little high. He has no complaints, takes no medications, tries to adhere to a healthy diet, and rarely exercises. He reports that over the previous 5 years he has gained 6.8 kg (15 lb). Review of systems is otherwise non-contributory. Physical examination is notable for obesity and blood pressure 172/86 mmHg. The remainder of the examination is unremarkable.

A

HT

236
Q

A 31-year-old woman presents with a 1-week history of fever, chills, fatigue, and unilateral ankle pain. Her past medical history includes mitral valve prolapse and hypothyroidism. She admits to infrequent intravenous heroin use and has a 10-pack-year history of smoking. Physical examination reveals temperature of 39°C (102°F), regular heart rate 110 beats per minute, blood pressure 110/70 mmHg, and respiration rate of 16 breaths per minute. Her cardiovascular examination reveals a grade 2/4 holosystolic murmur that is loudest at the right upper sternal border. Her right ankle appears red and warm, and is very painful on dorsiflexion.

A

infective endocarditis

237
Q

A 54-year-old man with a medical history of hypertension, diabetes, dyslipidaemia, smoking, and family history of premature coronary artery disease presents with retrosternal crushing chest pain (10/10 in intensity), radiating down the left arm and left side of the neck. He feels nauseated and light-headed and is short of breath. Examination reveals a hypotensive, diaphoretic man in considerable discomfort with diffuse bilateral rales on chest auscultation. ECG reveals convex ST-segment elevation in leads V1 to V6.

A

STEMI

238
Q

A 70-year-old woman is 2 days post-operative for knee replacement surgery. Her past medical history includes type 2 diabetes and a 40 pack-year history of smoking. She reports feeling suddenly unwell with dizziness, nausea, and vomiting. She denies any chest pain. On examination she is hypotensive and diaphoretic. ECG shows convex ST-segment elevation in leads II, III, and aVF with reciprocal ST segment depression and T-wave inversion in leads I and aVL.

A

STEMI

239
Q

A 65-year-old man, who smokes and has a history of hypertension and peripheral vascular disease, now presents with increasing frequency and severity of chest discomfort over the past week. He reports that he previously had chest pain after walking 100 metres, but now is unable to walk more than 50 m without developing symptoms. The pain radiates to the left side of the neck and is only eased after increasing periods of rest.

A

unstable angina

240
Q

A 45-year-old woman, with a history of type 1 diabetes diagnosed when she was a teenager, presents to the accident and emergency department complaining of abdominal pain, nausea, and shortness of breath that woke her up from sleep.

A

unstable angina

241
Q

A 50-year-old man presents to clinic with a complaint of central chest discomfort of 2 weeks’ duration, occurring after walking for more than 5 minutes or climbing more than 1 flight of stairs. The chest discomfort resolves with rest within several minutes. He is obese, has a history of hypertension, and smokes 10 cigarettes a day. His father died from a myocardial infarction at the age of 54 years. On examination, his blood pressure is 144/92 mmHg with a heart rate of 82 bpm. The remainder of his examination is normal.

A

stable angina

242
Q

A 60-year-old man with a history of a myocardial infarction presents to clinic for follow-up. He was started on aspirin, beta-blocker, and statin therapy after his heart attack. In the past 2 weeks the patient has noted return of chest pressure when he walks rapidly. The chest pressure resolves with sublingual glyceryl trinitrate or a decrease in his activity level. He is a former smoker and has modified his diet and activity to achieve his goal body weight. He is normotensive on examination with a heart rate of 72 bpm. The remainder of his examination is normal.

A

stable angina

243
Q

A 52-year-old woman presents with dyspnoea on exertion, fatigue, and occasional palpitations. She has no prior cardiac history. She denies chest pain, orthopnoea, paroxysmal nocturnal dyspnoea, or lower extremity oedema. On physical examination her jugular venous distension is around 12 cm and her lungs are clear to auscultation. Cardiac examination reveals a slightly displaced apical impulse with a palpable P2. Cardiac auscultation reveals III/VI holosystolic murmur at the apex that radiates to the axilla with diminished S1 and P2 greater than A2.

A

MR

244
Q

A 52-year-old woman presents with gradually increasing dyspnoea on exertion over the past 2 years. Recently she has required 2 pillows at night to alleviate recumbent dyspnoea. On examination, she has an apical diastolic murmur.

A

MS

245
Q

A 36-year-old prima gravida presents with dyspnoea on exertion and 2 pillow orthopnoea during her second trimester. Previous physical examinations had disclosed no cardiac abnormalities. On current physical examination, she has a loud S1 and a 2/6 diastolic rumble.

A

MS

246
Q

A 36-year-old woman presents with a 6-month history of gradually progressive dyspnoea on exertion and fatigue. On physical examination, her vital signs are normal and she appears not to be in any distress. Her lungs are clear to auscultation. Her cardiac examination shows a prominent jugular V wave, an accentuated pulmonic component to the second heart sound (P2), and a high-pitched holosystolic murmur best heard at the left sternal border.

A

pulmonary HT

247
Q

A 45-year-old woman presents with complaints of heaviness and fatigue in her legs. She does not experience the symptoms when she first awakens, but they become more noticeable and prominent as the day progresses and with prolonged standing. When she is standing for most of the day she notes swelling in both legs. The symptoms are concentrated over her medial calf, where she has prominent tortuous veins. She first noted dilated veins about 20 years ago when she was pregnant. Initially they did not cause her any discomfort but they have progressively enlarged and over the past 10 years have become increasingly painful. She recalls that her mother had similar veins in her legs.

A

varicose veins

248
Q

A 60-year-old man presents with syncope while walking outside with his wife. His wife recalls the patient looking at the sky to point out an aeroplane. He then appeared pale and collapsed to the ground, suffering a head laceration in the process. The history suggested the possibility of carotid sinus syndrome (CSS). In the laboratory, carotid sinus massage was undertaken while the patient was in the upright posture on a tilt table. The massage induced 10 seconds of asystole with near loss of consciousness. A dual-chamber pacemaker was implanted on the basis of a presumptive diagnosis of CSS; there was no recurrence of syncope during the next year. This patient has had no further faints but occasionally becomes dizzy when turning his head abruptly (CSS).

A

vasovagal syncope

249
Q

An 18-year-old man presents to a clinic reporting 2 episodes of loss of consciousness. The first episode occurred 1 year earlier while playing dodge ball in gym class. He recalls diving to the ground to avoid being hit. On getting up quickly, he noticed feeling lightheaded, sweaty, and nauseated. Apparently, he fell to the ground but does not recall having done so. He later recalls waking up in an accident and emergency (A&E) department. Witnesses reported shaking and clenching of both hands after he had fallen. In the A&E department he was given phenytoin intravenously because of concern that he may have had a seizure. After a negative work-up in the hospital he was prescribed carbamazepine, despite no abnormalities on an electroencephalogram. A year later he had a second episode of loss of consciousness while doing bicep curls. He denies palpitations, tongue biting, or incontinence. He experienced a similar prodrome of warmth and lightheadedness. He has no history of seizures outside of these 2 episodes

A

vasovagal syncope

250
Q

A 63-year-old man with sun-damaged skin presents with a small nodule on the left aspect of his forehead. He mentions that it is itchy at times, and he thinks that he may have seen a colleague 2 years previously for removal of some keratoses or scabs. The patient indicates that these were either cauterised or frozen. On examination there is a pearly white nodule with prominent telangiectasia on its surface.

A

BCC

251
Q

A woman in her mid-40s with dark, leathery skin and intense wrinkling of the lower neck (signifying excessive sun exposure either in a form of frequent sun tanning beds or perhaps frequent beach visits) presents at your office. She reports she has had multiple facial lifts, to decrease wrinkles. The plastic surgeon she has consulted performed other cosmetic procedures, including botulinum toxin type A injections. She complains about a mole on her jaw that has recently started to bleed.

A

BCC

252
Q

A 28-year-old man presents with pain on swallowing. He has no oral symptoms, but clinically has abundant, creamy white, loosely adherent plaques throughout his mouth. Lesions are especially prominent in his buccal, palatal, and pharyngeal mucosa. HIV infection was diagnosed 2 years ago, but he has not yet started anti-retroviral treatment. His last CD4 count and viral load measurement was 8 months ago.

A

oral candidiasis

253
Q

A 64-year-old man presents with a complaint of burning under his maxillary denture. He has hypertension and osteoarthritis. His medications include a thiazide diuretic, a non-selective beta-blocker, and an OTC analgesic. Intra-orally, he has severely erythematous palatal mucosa, with a distinct granular appearance. His mucosa is dry and his salivary flow is minimal.

A

oral candidiasis

254
Q

A 45-year-old man presents with acute onset of pain and redness of the skin of his lower leg. Low-grade fever is present and the pretibial area is erythematous, oedematous, and tender.

A

cellulitis

255
Q

A 12-year-old female presents with dry, itchy skin that involves the flexures in front of her elbows, behind her knees and in front of her ankles. Her cheeks also have patches of dry, scaly skin. She has symptoms of hay fever and has recently been diagnosed with egg and milk allergy. She has a brother with asthma and an uncle and several cousins who have been diagnosed with eczema.

A

eczema

256
Q

An otherwise healthy, white, 10-year-old boy complains of a sore throat. He is running a low-grade fever of 38.0°C (100.6°F). Red lesions have developed on his extremities, and he is having some difficulty breathing after exertion. Oral erosions have limited his food and fluid intake. On physical examination, there are target lesions on his upper and lower extremities with erosions on his lower lip and palate. High-pitched wheezing is heard over the lower lungs bilaterally, and the patient is in mild discomfort when sitting upright. He is unable to open his mouth fully due to crusted oral lesions.

A

erythema multiforme

257
Q

A 25-year-old otherwise healthy woman awakens with a mild sore throat, fatigue, pain in both ankles, and red lesions on her legs. She reports weakness and fatigue for the past 2 days, but denies fever, chills, nausea, or night sweats. There is no history of using recreational drugs, oral contraceptives, or other medications. She has not travelled abroad for 3 years. Physical examination reveals red, tender, fixed, deep-seated nodules on both shins. Her ankles are mildly swollen and tender and she reports difficulty bearing weight. Chest, cardiovascular and ophthalmological examinations are normal. A chest x-ray shows left lower lobe infiltration. Her tuberculin skin test is negative. She is diagnosed with pneumococcal pneumonia.

A

erythema nodosum

258
Q

A 28-year-old woman awakens with red eyes, photophobia, and nodules on her legs. She denies fever, malaise, cough, or joint pains. She enjoys dairy products, but denies recent consumption of raw milk or cheese. She does not take recreational drugs. Physical examination shows bilateral iritis and enlarged lachrymal glands. Chest and cardiovascular examinations are normal, though the skin examination reveals many dark red nodules on her legs and lower thighs. These skin nodules are tender and non-mobile. Her tuberculin test is negative, but a chest x-ray shows bilateral hilar adenopathy consistent with the diagnosis of sarcoidosis.

A

erythema nodosum

259
Q

A 55-year-old woman presents with a right flank mass. She states she was recently diagnosed with diabetes mellitus, which she has been able to control with diet modifications. She lost 9 kg (20 pounds) within 3 months and then noticed a mass over her right lower rib cage. She denies pain but does report discomfort when she wears a jogging bra. On physical examination, the mass is soft, superficial, and mobile, and it measures 5 cm in diameter.

A

lipoma

260
Q

A 35-year-old man presents with a right thigh nodule and a recurrent left chest wall nodule at the site of a prior scar. He states that he noticed a bump on his right lateral thigh 2 years previously and that the left chest wall lesion had been removed in clinic 3 years prior. The nodules have grown slightly over recent months. He also states that they bother him when he touches them. On physical examination, the nodules are 1 cm x 2 cm, soft, and mobile, and they feel subcutaneous.

A

lipoma

261
Q

A sexually active female university student presents complaining of itchy growths on the pubic area and the inner thighs, of 1 month’s duration. The patient has been using a condom when she is sexually active with her boyfriend of 3 months. She is seen every 6 months by her gynaecologist and had a negative cervical smear 2 months ago. Pearly papules with a central dell can be observed on the pubic area and inner thighs. Some have surrounding erythema and excoriations.

A

molluschi contagiosum

262
Q

A 65-year-old man presents with fever and respiratory distress. He rapidly develops progressive hypoxia and hypotension requiring ventilator support and pressors. Efforts to reposition the patient result in marked desaturations. One week later, following stabilisation of his medical condition, he is noted to have an extensive area of tissue damage over his sacral region. The damage around the margin of the wound appears relatively superficial, with some signs of partial skin loss. Towards the centre, directly over the sacrum, is an area of necrotic tissue, which is starting to separate spontaneously, exposing a cavity containing viscous yellow slough. A purulent discharge is draining from this cavity.

A

pressure sore

263
Q

An 80-year-old woman in a residential care facility, who has recently suffered a stroke with a resulting right hemiplegia, presents with a painful area on right heel. On examination the heel is found to be covered with a layer of hard, black, necrotic tissue with a leathery appearance.

A

pressure sore

264
Q

A middle-aged man with a known history of psoriasis presents with white scaly papules and plaques on his elbows, extensor arms, knees, and shins. In the past 6 months, these lesions have become much worse and have started to appear on his waist and hip. Scaly and flaky eruptions are also present on his scalp, ears, and eyebrows. He describes the lesions as being itchy and irritating. He is a heavy smoker and has been unsuccessful in a previous attempt at smoking cessation.

A

psoriasis

265
Q

A 33-year-old woman with a past medical history of hypothyroidism presents with complaints of hives for the past 4 months. She describes red, raised, itchy lesions that involve her entire body, including her face. She also reports 2 episodes of face and tongue swelling, each of which prompted her to report to the nearest emergency department. In addition to itching, the lesions sometimes cause a burning sensation. The lesions and symptoms resolve over 24 to 36 hours. Despite countless attempts, she has not been able to associate the hives with any specific triggers. The patient voices extreme frustration and feelings of depression, which she attributes to her recent condition.

A

urticaria

266
Q

A 50-year-old man with a past medical history of HTN and a recent diagnosis of osteoarthritis presents to his primary care physician with complaints of hives over the past 2 weeks. He reports red and raised lesions that are intensely pruritic and involve his torso and bilateral extremities. He denies any swelling or pain associated with the episodes. The patient also denies any unusual food ingestions or recent changes in his environment (e.g., soaps, detergents). However, he has recently started using scheduled ibuprofen for osteoarthritis.

A

urticaria

267
Q

A 6-year-old boy presents with fever, headache, and a diffuse, pruritic, vesicular rash, which is most prominent on the face and chest. He has had generalised malaise and low-grade fever for a few days prior to presentation. He developed high fever and a rash in the last 48 hours. Physical examination demonstrates a temperature of 39°C (102°F) and heart rate of 140 beats/minute. He has a few scattered vesicular lesions in his oropharynx and his lung fields are clear. The lesions are prominent on the face and chest, but all extremities are also involved. In some areas the lesions are crusted, while in others they appear newly formed. He has no nuchal rigidity or other meningeal signs. The child has never been immunised for varicella, and a classmate at his school had chickenpox a few weeks ago.

A

VZV

268
Q

A 42-year-old man is referred to the liver clinic with mild elevation in alkaline phosphatase and aminotransferases for several years. He has a medical history significant for obesity, hypertension, and hypercholesterolaemia. He does not smoke or drink alcohol and there is no high-risk behaviour. He has a family history of premature cardiac disease. He is taking a diuretic and discontinued a statin several months ago. Other than complaints of mild fatigue, the patient feels well. Examination is notable for a BMI of 37 kg/m^2, truncal obesity, and mild hepatomegaly.

A

NASH

269
Q

A 63-year-old woman is admitted to the hospital with new-onset ascites. She has a history of long-standing diabetes and hypertension. She has never formally been given a diagnosis of liver disease. Despite increasing abdominal distension, she has lost 13.5 kg in the last year. Physical examination reveals a lethargic-appearing woman with temporal wasting, massive ascites, and 2+ pitting oedema. She has numerous spider nevi over her chest wall and marked palmar erythema.

A

NASH

270
Q

A 55-year-old man presents with severe dysphagia to solids and worsening dysphagia to liquids. His social history is significant for 40 pack-year cigarette smoking and a 6-pack of beer per day. He has lost over 10% of his body weight and currently is nourished only by milkshake supplements. He complains of some mild odynophagia and is constantly coughing up mucus secretions.

A

oesophageal cancer

271
Q

A 41-year-old alcoholic man has a 6-year history of recurrent attacks of pancreatitis characterised by epigastric pain radiating to the back. The initial attack required hospitalisation for severe pain, and clinical chemistry showed a >15-fold elevation in serum amylase and lipase. Subsequent attacks were less severe, managed primarily as an outpatient, and lasted less than 10 days, with long symptom-free intervals. After detoxification 6 months ago he had no further attacks, but has recently developed evidence of diabetes and steatorrhoea. Computed tomography imaging shows pancreatic calcifications but no cystic or mass lesions.

A

chronic pancreatitis

272
Q

A 20-year-old man reports chronic offensive discharge of pus and blood from his natal cleft. He relates that several months previously he had a ‘boil’ in the same region, which was painful and then burst spontaneously. On examination, the patient has dense body hair. There are several sinus tracts in the midline or just lateral to the natal cleft, from which offensive-smelling discharge can be expressed.

A

pilonidal sinus

273
Q

A 55-year-old male presents with several episodes of haematemesis in the past 24 hours. He has a history of alcoholic cirrhosis and is being treated for ascites with diuretics and for encephalopathy with lactulose. Currently he is confused and unable to give a complete history. His vital signs include a pulse of 85 bpm and BP of 84/62 mmHg. He is noted to have jaundice, splenomegaly, and multiple spider angiomas.

A

oesophageal varices

274
Q

A 50-year-old woman undergoing health screening is found to have a cholestatic pattern on her LFT results. Her alkaline phosphatase and gamma-GT concentrations are elevated, although transaminases, bilirubin, and albumin concentrations are normal. On questioning she mentions that she had been getting increasingly tired over the past few years but felt that this was simply a result of her age and work pattern. She also describes occasional itch that feels as if it is deep underneath the skin and that is not associated with a rash. She had no other past medical history but had a family member who had autoimmune thyroid disease. Clinical examination reveals no abnormal findings other than excoriations related to itch and xanthelasmata around the eyes.

A

PBC
Although most primary biliary cholangitis (PBC) patients present early in the disease process with abnormal liver biochemistry (with or without the symptoms of itch or fatigue), occasionally patients will present with advanced liver disease. In these patients, the clinical features of cirrhosis would be more prominent than the features of PBC. Possible features would include ascites, splenomegaly, skin thinning, weight loss, and variceal bleeding. Jaundice may be significantly more prominent than would be expected in people with cirrhosis of different aetiologies. The disease can present from the 20s onwards and there is increasing evidence to suggest that it is more severe in nature and less responsive to first-line therapy in younger patients.

Use of this content is s

275
Q

A 43-year-old man with a history of mild ulcerative colitis is noted to have an elevated serum alkaline phosphatase, slightly elevated aminotransferases, and normal bilirubin on routine laboratory testing. He complains of fatigue and upper abdominal pain. He denies pruritus or fevers. Physical examination is unremarkable.

A

PSC

276
Q

A 27-year-old man with a 3-month history of rectal bleeding and diarrhoea is referred for evaluation. Laboratory tests show mild anaemia, a slightly elevated sedimentation rate, and the presence of white blood cells in stool. Stool culture is negative. Colonoscopy shows continuous active inflammation with loss of vascular pattern and friability from the anal verge up to 35 cm, with a sharp cut-off. The colonic mucosa above 35 cm appears normal, as does the terminal ileum. Biopsy specimens show active chronic colitis.

A

UC

277
Q

A 34-year-old man presents 2 weeks after returning from a month-long trip to India. He denies attending pre-travel vaccination clinic and did not take prophylaxis of any sort while in India. He reports a 6-day history of malaise, anorexia, abdominal pain, nausea with emesis, and dark urine. He admits to dietary indiscretion and consumed salad at a road-side vendor 3 weeks before onset of symptoms. On examination there is icterus. His alanine transaminase (ALT) is 5660 units/L, and total bilirubin 153.9 micromols/L (9 mg/dL). Serum IgM anti-hepatitis A virus (HAV) antibodies are detected.

A

hep A

278
Q

A 40-year-old asymptomatic man presents for a routine visit with elevated alanine aminotransferase (ALT) level (55 international units [IU]/mL). His mother died of hepatocellular carcinoma and he has a middle-aged sister with “hepatitis B infection”. He has a normal physical examination and has no stigmata of chronic liver disease.

A

hep b

279
Q

A 62-year-old man presents for a routine initial visit in New York. He has occasional arthralgia or myalgia, and takes an ACE inhibitor and a thiazide diuretic for hypertension. A retired accountant and non-smoker, he drinks 1 or 2 beers per week and denies current drug use. Physical examination is normal except for being overweight. Routine blood chemistries are normal, but a screening hepatitis C virus (HCV) antibody test is positive. At follow-up, the patient is concerned about whether he will develop liver problems. He had heard on television that new oral medications are easier to take than older regimens that used injections and asks about the next diagnostic and treatment steps.

A

Hep C

280
Q

A 60-year-old man presents with several months of gradually worsening abdominal swelling, intermittent haematemesis, and dark stool. He denies chest pain or difficulty breathing. Past medical and family history are not contributory. Past surgical history is significant for back surgery requiring blood transfusion in 1990. Social history is significant for occasional alcohol use. BP is 110/80 mmHg. Physical examination is significant for spider angiomata on the upper chest, gynaecomastia, caput medusae, and a fluid wave of the abdomen. The rest of the examination is normal.

A

Hep C

281
Q

A 20-year-old previously healthy man has an episode of jaundice associated with tiredness. He is found to have elevated serum aminotransferase enzymes (AST 150 units/L, ALT 175 units/L) and elevated serum bilirubin 77.0 micromols/L (4.5 mg/dL). Ophthalmological examination for Kayser-Fleischer rings is negative.

A

Wilsons

282
Q

An 18-year-old woman presents with bilateral tremor of the hands. She is a senior in high school and during the year her grades have plummeted to the point that she is failing. She says her memory is now poor, and she has trouble focusing on tasks. Her behaviour has changed in the past 6 months in that she has frequent episodes of depression, separated by episodes of bizarre behaviour, including shoplifting and excessive drinking. Her parents and other authorities have begun to suspect her of using street drugs, which she denies. Her handwriting has become very sloppy. Her parents have noted slight slurring of her speech. Physical examination reveals upper extremity tremor, mild dystonia of the upper extremities and mild incoordination involving her hands. Slit-lamp examination reveals Kayser-Fleischer rings.

A

Wilsons

283
Q

A 67-year-old retired construction worker has shortness of breath with activity that has been gradually getting worse, and a chronic cough. He denies chest pain. He has a 45-pack-year smoking history, but stopped smoking aged 50 years. There is no family history of lung disease. He does not take any respiratory medicine on a regular basis. With colds he has noticed wheezing and his doctor once prescribed an inhaler.

A

Asbestosis

284
Q

A 55-year-old factory maintenance worker falls at work. A CXR is performed to evaluate the patient for a possible broken rib. Bilateral pleural thickening is seen on CXR. Further history indicates he is very active without any respiratory symptoms. He smokes 20 cigarettes a day. There is no family history of lung disease. He does not take any respiratory medicine.

A

asbestosis

285
Q

A 72-year-old man presents to his primary care physician with a history of increasing shortness of breath over a period of several months. Before his retirement he was a construction worker. Physical examination reveals decreased breath sounds in the right lung base associated with dullness to percussion.

A

mesothelioma

286
Q

A 65-year-old man underwent induction chemotherapy for recently diagnosed acute myelogenous leukaemia. Antimicrobial prophylaxis included norfloxacin, fluconazole, and aciclovir. During chemotherapy-induced neutropenia, he received empirical antibiotic therapy for the fever without an obvious source of infection. Blood cultures were negative and fever subsided. During the third week of neutropenia, fever recurred with dry cough and left-sided pleuritic pain. Physical examination demonstrated no significant abnormalities. Blood cultures remained negative. CXR was normal. However, a high-resolution CT scan of his chest revealed a 2 cm peripheral nodule with a surrounding ‘halo’ sign in the left upper lobe.

A

aspergillosis

287
Q

A 67-year-old man with COPD presents with recent changes in his CXR. He has shortness of breath that has not changed from his baseline status. On examination, he is afebrile with clinical evidence of chronic lung disease. The CXR reveals a right upper lobe cavitary lesion with an intracavitary mass and adjacent pleural thickening.

A

aspergillosis

288
Q

A 25-year-old man with asthma presents with complaints of cough, fever, and expectoration of brown mucus for 3 months. Although he has known allergic rhinitis and asthma, he feels his symptoms were previously well controlled on his regimen of oral antihistamines and inhaled corticosteroids. His exam is significant for end-expiratory wheezing.

A

ABPA

289
Q

A 16-year-old girl with known cystic fibrosis presents to the ER with an acute exacerbation of her chronic cough and wheezing. She has noticed an increase in sputum production and decreased exercise tolerance.

A

ABPA

290
Q

A 25-year-old woman presents with shortness of breath. She reported that in high school, she occasionally had shortness of breath and would wheeze after running. She experiences the same symptoms when she visits her friend who has a cat. Her symptoms have progressively worsened over the past year and are now a constant occurrence. She also finds herself wheezing when waking from sleep approximately twice a week.

A

asthma

291
Q

A 55-year-old woman presents for evaluation of a chronic cough, productive of thick, yellow sputum that sometimes becomes blood-tinged. She has experienced recurrent episodes of fever associated with pleuritic chest pain. She states that she is embarrassed by the persistent, intractable nature of her cough and has been prescribed multiple courses of antibiotics. Over the last 5 years, she has developed shortness of breath with exertion. Her past medical history is significant for pneumonia as a child and sinus polyps during adulthood for which she has had surgery.

A

bronchiectasis

292
Q

A 66-year-old man with a smoking history of one pack per day for the past 47 years presents with progressive shortness of breath and chronic cough, productive of yellowish sputum, for the past 2 years. On examination he appears cachectic and in moderate respiratory distress, especially after walking to the examination room, and has pursed-lip breathing. His neck veins are mildly distended. Lung examination reveals a barrel chest and poor air entry bilaterally, with moderate inspiratory and expiratory wheezing. Heart and abdominal examination are within normal limits. Lower extremities exhibit scant pitting oedema.

A

COPD

293
Q

A 56-year-old woman with a history of smoking presents to her primary care physician with shortness of breath and cough for several days. Her symptoms began 3 days ago with rhinorrhoea. She reports a chronic morning cough productive of white sputum, which has increased over the past 2 days. She has had similar episodes each winter for the past 4 years. She has smoked 1 to 2 packs of cigarettes per day for 40 years and continues to smoke. She denies haemoptysis, chills, or weight loss and has not received any relief from over-the-counter cough preparations.

A

COPD

294
Q

A 38-year-old man presents with fever of 38.5°C (101.2°F), chills, myalgias, non-productive cough, and dyspnoea. Other than tachypnoea, tachycardia, and bibasilar rales, the rest of the physical examination is normal. He reports that this happens almost every month the day after he cleans out the bird cages in which he keeps the pigeons that he breeds and races.

A

extrinsic allergic alveoli’s

295
Q

A 65-year-old man presents with gradually progressive dyspnoea on exertion and a non-productive cough. He has no history of underlying lung disease and no features that would suggest an alternative aetiology for his cough and dyspnoea. He has no history of joint inflammation, skin rashes, or other features of a systemic inflammatory disease such as lupus or rheumatoid arthritis. He is on no medications and has no environmental exposures to organic allergens such as mould. On examination, he has fine crackles audible over his lung bases bilaterally but no evidence of volume overload. He has clubbing of his fingers.

A

IPF

296
Q

A 72-year-old man with a history of cigarette smoking presents with mild shortness of breath. He is treated initially with inhaled bronchodilators for a presumed diagnosis of chronic obstructive lung disease but has no symptomatic improvement. PFTs are performed and show restriction rather than obstruction, and impaired diffusing capacity for carbon monoxide. A follow-up CXR shows prominent bi-basilar interstitial markings.

A

IPF

297
Q

A 65-year-old man presents with a 2-month history of a dry persistent cough and 4.5 kg unintentional weight loss. He denies fevers, dyspnoea, sore throat, rhinorrhoea, chest pain, or haemoptysis. Past medical history is significant for chronic obstructive pulmonary disease and hypertension. Family history is non-contributory. He smoked 1 pack of cigarettes daily for 40 years but quit 5 years ago. No adenopathy was palpable on examination and breath sounds were diminished globally without focal wheezes or rales.

A

SCLC

298
Q

A 65-year-old man presents with a 2-month history of a dry persistent cough and 4.5 kg unintentional weight loss. He denies fevers, dyspnoea, sore throat, rhinorrhoea, chest pain or haemoptysis. Medical history is significant for COPD and hypertension. Family history is non-contributory. He smoked 1 pack of cigarettes daily for 40 years but quit 5 years ago. No adenopathy is palpable on examination and breath sounds are diminished globally without focal wheezes or crackles.

A

NSCLC

299
Q

A 41-year-old obese man presents with loud chronic snoring and gasping episodes during sleep. His wife has witnessed episodic apnoea. He reports unrefreshing sleep, multiple awakenings from sleep, and morning headaches. He has excessive daytime sleepiness, which is interfering with his daily activities, and he narrowly avoided being involved in a motor vehicle accident. His memory is also affected. He has been treated for hypertension, gastro-oesophageal reflux, and type 2 diabetes.

A

OSA

300
Q

A 76-year-old retired steelworker has shortness of breath with activity that has been gradually getting worse, and a chronic cough. He denies chest pain. He has a 45-pack/year smoking history, but stopped aged 50. There is no family history of lung disease. He does not take any respiratory medication on a regular basis. He has noticed that he wheezes when he has an upper respiratory infection (URI), and his doctor once prescribed him an inhaler. He is also bothered by joint swelling and stiffness. Lung auscultation is normal.

A

pneumoconiosis

This case is a common clinical presentation of silicosis or coal workers’ pneumoconiosis.

301
Q

A 35-year-old man who works machining beryllium-copper alloy for the electronics industry is concerned about the possibility of adverse health effects from beryllium, which is a component of the metal he is machining. He has heard about a blood test that can be used for diagnosing beryllium disease. He is not sure if he has had some increased shortness of breath with exercise. He has never smoked cigarettes. He has no personal or family history of allergies or asthma. Lung auscultation is normal.

A

(This case is a common clinical presentation of chronic beryllium disease.)

302
Q

A 54-year-old smoker with multiple comorbidities (diabetes, hypertension, coronary artery disease) presents with a 2-day history of a productive cough with yellow sputum, chest tightness, and fever. Physical examination reveals a temperature of 38.3°C (101°F), BP of 150/95 mmHg, heart rate of 85 bpm, and a respiratory rate of 20 breaths per minute. His oxygen saturation is 95% at rest; lung sounds are distant but clear, with crackles at the left base. CXR reveals a left lower lobe infiltrate.

A

CAP
Pneumonia can occur at any age, but the incidence increases significantly in old age, and pneumonia is a leading cause of illness and death in older patients. The clinical manifestations of pneumonia in elderly persons are often less intense than those in younger patients. [1] Atypical pathogens such as Mycoplasma pneumoniae , Chlamydophila pneumophila , and respiratory viruses can present in a subacute fashion with gradual onset of fever, non-productive cough, constitutional symptoms, relatively normal white blood cell count, and absent or diffuse findings on lung examination. [2] Patients with severe pneumococcal or Legionella pneumophila pneumonia often progress rapidly to respiratory failure.

303
Q

A 55-year-old man with a history of peripheral vascular disease, who presents with a complaint of a left foot ulcer and pain when walking short distances, is found to have a popliteal stenosis and admitted for re-vascularisation. Four days after admission, on postoperative day 3, he develops SOB, hypoxia, and a productive cough. Auscultation of his chest reveals decreased breath sounds at the lower aspect of the right side of his chest. His morning leukocyte count is slightly higher than the day before at 11,000 cells/mL^3. An anterior-posterior bedside CXR reveals right lower lobe opacity.

A

HAP

304
Q

An 88-year-old female resident of a nursing home, who typically does not present to the acute care hospital, has frequent UTIs that are managed by the nursing home physician. In the nursing home, she develops a UTI due to multi-drug-resistant pathogens. On admission to hospital, she has poor mental status and her bed is left with the head elevated to only a 5° angle. On hospital day 4, a CXR reveals a right lower lobe opacity.

A

HAP

305
Q

A 20-year-old man presents to the emergency department with complaints of left-sided chest pain and shortness of breath. He states that these symptoms began suddenly 4 days ago while he was working at his computer. He initially thought that he might have strained a chest wall muscle, but because the pain and dyspnoea had not resolved, he decided to seek medical attention. He has no significant past medical history but has smoked cigarettes since the age of 16 years. His older brother suffered a pneumothorax at the age of 23 years. The patient’s vital signs are normal. He appears in mild discomfort. Examination of his chest reveals that the left hemithorax is mildly hyperexpanded with decreased chest excursion. His left hemithorax is hyper-resonant on percussion, and breath sounds are diminished when compared with the right hemithorax. His cardiovascular examination is normal.

A

pneumothorax

306
Q

A 65-year-old patient with COPD presents to the emergency department with complaints of worsening shortness of breath and right-sided chest discomfort. He states that these symptoms occurred suddenly 1 hour prior to presentation. He denies fevers and chills. He also denies increased sputum production and a change in the colour or character of his sputum. He continues to smoke cigarettes against medical advice. The patient’s blood pressure is 136/92 mmHg, heart rate is 110 beats per minute, and respiratory rate is 24 breaths per minute. Chest excursion is decreased on the right more than the left. His right hemithorax is more hyperinflated than the left. His right hemithorax is hyper-resonant on percussion. Breath sounds are distant bilaterally but more diminished on the right.

A

pneumothorax

307
Q

A 65-year-old man presents to the emergency department with acute onset of shortness of breath of 30 minutes’ duration. Initially, he felt faint but did not lose consciousness. He is complaining of left-sided chest pain that worsens on deep inspiration. He has no history of cardiopulmonary disease. A week ago he underwent a total left hip replacement and, following discharge, was on bed rest for 3 days due to poorly controlled pain. He subsequently noticed swelling in his left calf, which is tender on examination. His current vital signs reveal a fever of 38.0°C (100.4°F), heart rate 112 bpm, BP 95/65 mmHg, and an O₂ saturation on room air of 91%.

A

PE

308
Q

A 29-year-old woman presents with shortness of breath, cough, and painful red skin lesions on the anterior surface of the lower part of both legs. CXR reveals bilateral hilar lymphadenopathy with pulmonary infiltrates.

A

sarcoidosis

309
Q

A 35-year-old woman presents with skin lesions around her nose, which are indurated plaques with discoloration. She also reports a red, moderately painful right eye with blurred vision and photophobia.

A

sarcoidosis

310
Q

A 34-year-old man presents to his primary care physician with a 7-week history of cough that he describes as non-productive. He has had a poor appetite during this time and notes that his clothes are loose on him. He has felt febrile at times, but has not measured his temperature. He denies dyspnoea or haemoptysis. He is originally from the Philippines. He denies any history of TB or TB exposure. Physical examination reveals a thin, tired-appearing man but is otherwise unremarkable.

A

TB

311
Q

A 70-year-old woman with a history of hypertension, diabetes mellitus, hyperlipidaemia, and prior myocardial infarction presents to the emergency department with palpitations and shortness of breath. These symptoms started 2 days ago. She was diagnosed to have AF with rapid ventricular rate response a year and a half ago, at which time an attempted direct current cardioversion and a trial of sotalol to maintain sinus rhythm and prevent further episodes of AF were unsuccessful. The patient was treated with digoxin and metoprolol to control rate and warfarin to prevent stroke. Current physical examination shows that she is febrile and has an irregularly irregular radial pulse at a rate between 90 and 110 bpm, BP 100/70 mmHg, and respiratory rate of 26 breaths per minute. Heart sounds are irregular, but no S3 or S4 gallop or murmurs are audible. The breath sounds are of bronchial character associated with crepitations over left basal lung area.

A

chronic AF

Can you list out treatment?